Download ID_3965_Infectious diseases test_English_sem_9

Document related concepts

Dental emergency wikipedia , lookup

Eradication of infectious diseases wikipedia , lookup

Transmission (medicine) wikipedia , lookup

Focal infection theory wikipedia , lookup

Syndemic wikipedia , lookup

Compartmental models in epidemiology wikipedia , lookup

Canine parvovirus wikipedia , lookup

Pandemic wikipedia , lookup

Marburg virus disease wikipedia , lookup

Infection wikipedia , lookup

List of medical mnemonics wikipedia , lookup

Infection control wikipedia , lookup

Transcript
1.
A.
B.
C.
D.
E. *
2.
A. *
B.
C.
D.
E.
3.
A.
B.
C.
D.
E. *
4.
A.
B.
C.
D.
E. *
5.
A. *
B.
C.
D.
E.
6.
A.
B. *
C.
D.
E.
7.
A. *
B.
C.
D.
E.
A suspected case of typhoid fever of 1st week is admitted in the hospital. What specific examination
(laboratory diagnosis) do you suggest for this patient?
Biochemical blood analysis
Reaction of agglutination of Vidall
X-ray
Urinoculture
Hemoculture
A suspected case of typhoid fever of 1st week is admitted into the hospital. What examination
(laboratory investigation) do you prescribe for this patient?
Hemoculture
Reaction of agglutination of Vidall
Indirect hemaglutination test with О-, Н-, Vi-аntigens
Urinoculture
Coproculture
A suspected case of typhoid fever of 3-d week is admitted in the hospital. What examination
(laboratory investigation) do you prescribe for this patient?
Coproculture
Reaction of agglutination of Vidall
Urinoculture
Hemoculture
All enumerated
A suspected case of typhoid fever of 2-nd week is admitted in the hospital. What examination
(laboratory diagnosis) do you suggest for this patient?
Coproculture
Reaction of agglutination of Vidall
Urinoculture
Hemoculture
All enumerated
About what does the diagnostic titre of reaction of Vі-haemaglutination testify?
About typhoid bacterial-carrier
About the period of height of the epidemic typhus
About meningococcaemia
About a malaria
About the latent period of brucellosis
After salmonellosis chronic carrier state.
Not formed
Formed in 0,1-1 % of the cases
Formed in 8-10 % of cases
Formed in 20-30 % of cases
Formed in 50-80 % of cases
All laboratory and instrumental examinations are needed to confirm the diagnosis of food poisoning,
except:
Blood analysis
Coproculture
Investigation of washing waters
Stool test
Serum researches with the autoculture of substance
8.
A. *
B.
C.
D.
E.
9.
A. *
B.
C.
D.
E.
10.
A.
B. *
C.
D.
E.
11.
A.
B. *
C.
D.
E.
12.
A.
B. *
C.
D.
E.
13.
A. *
B.
C.
D.
E.
14.
A.
B.
All laboratory and instrumental examinations are needed to confirm the diagnosis of food poisoning,
except:
Urinoculture
Coproculture
Investigation of washing waters
Stool test
Serum researches with the autoculture of substance
All the laboratory and instrumental examinations are needed to confirming the diagnosis of food
poisoning, except:
General blood analysis
Coproculture
Stool test
Food debris
Serum researches with the autoculture of substance
At the typical form of typhoid fever temperature of body rises to 39-40 °C. The temperature curve
likes to 2 waives (during 3-4 weeks of disease). How is such temperature curve named?
The temperature curve as Vunderlihs
The temperature curve as Botkin
Temperature curve as Kildushevsky
Temperature curve as Ellers
Intermittent temperature curve
At the typical form of typhoid fever temperature of body rises to 39-40 °C. The temperature curve
likes to 2 waives (during 3-4 weeks of disease). How is such temperature curve named?
Febris intermittent
The temperature curve as Botkin
Febris recurrent
Temperature curve as Kildushevsky
Normal temperature
At the typical form of typhoid fever temperature of body rises to 37-37,5 °C. How is such
temperature curve named?
The temperature curve as Vunderlihs
The subfebrile temperature curve
Temperature curve as Kildushevsky
Temperature curve as Ellers
Intermedium temperature curve
At the typical form of typhoid fever, temperature of body rises like to stairs from a day to the day to
39-40 °C.
The temperature curve as Vunderlihs
The temperature curve as Botkin
Temperature curve as Kildushevsky
Temperature curve as Ellers
Intermedium temperature curve
At the typical form of typhoid fever, temperature of body rises like to stairs from a day to the day to
39-40 °C at the end of 1st week, and then slowly during 2-3 weeks gradually going down, comes to
the norm. How is such temperature curve named?
The temperature curve as Vunderlihs
The temperature curve as Botkin
C. *
D.
E.
15.
A.
B. *
C.
D.
E.
16.
A.
B.
C.
D.
E. *
17.
A.
B.
C.
D. *
E.
18.
A.
B.
C.
D. *
E.
19.
A.
B.
C.
D.
E. *
20.
A. *
B.
C.
D.
E.
21.
A.
B. *
C.
D.
Temperature curve as Kildushevsky
Temperature curve as Ellers
Intermedium temperature curve
Bacteria typhoid, painted by Gram, look like.
The blue-violet rods
Red rods
Blue-violet cocci
Red cocci
Brown vibrio
Bacteria typhoid, painted by Gram, looks like:
The yellow rods
Black vibrio
Blue cocci
Red cocci
Red rods
Bacteria typhoid, painted by Gram, looks like:
Pink protozoa
Red fungi
Violet cocci
Red rods
Green vibrio
By which method is it possible to diagnose typhoid fever?
All enumerated
Coproculture
Bilicultura
Hemocultura
Reaction of Vidal
By which method is it possible to find out bacterial carrier in case of typhoid fever?
Coproculture
Reaction of agglutination of Vidall
Indirect hemaglutination test with О-аntigen
Indirect hemaglutination test with a Н-antigen
Indirect hemaglutination test with a Vi-antigen
By which method is it possible to find out bacterial carrier in case of typhoid fever?
Indirect hemaglutination test with a Vi-antigen
Reaction of agglutination of Vidall
Stool test
Indirect hemaglutination test with a Н-antigen
Swab from oropharynx
Convalescent of typhoid fever may be discharged from clinic after:
Non-permanent negative bacteriologic examination of feces
21-th day of normal temperature of body and 3-multiple negative bacteriologic examination of
excrements and urine
14-th day of normal temperature of body and 2-multiple negative bacteriologic examination of
excrements and urine
Clinical convalescence and normalization of rectoscopic picture
E.
22.
A. *
B.
C.
D.
E.
23.
A. *
B.
C.
D.
E.
24.
A.
B. *
C.
D.
E.
25.
A.
B.
C.
D. *
E.
26.
A.
B.
C.
D.
E. *
27.
A.
B.
C.
D.
E. *
28.
A. *
B.
C.
D.
Normalization of rectoscopic picture and decreasing of titre of antibodies in RNGA
Convalescent of typhoid fever may be discharged from clinic after:
21-th day of normal temperature of body and 3-multiple negative bacteriologic examination of
excrements and urine
Normalization of rectoscopic picture and decreasing of titre of antibodies in RNGA
3-th day of normal temperature of body and 2-multiple negative bacteriologic examination of
excrements and urine
Clinical convalescence and normalization of rectoscopic picture
14-th day of normal temperature of body and 2-multiple negative bacteriologic examination of
excrements and urine
District doctor suspected typhoid fever in a patient M., 15 y.o. For rendering harmless of patient as
the sources of infection it follows to conduct the following measures, except:
Introduction to the patient of antityphoid monovaccine
Hospitalization in the infectious department during 3-6 hours
Introduction etiotropic antibacterial treatment
Conducting of controls bacteriological researches of excrement and urine (triply) and bile before the
discharge
Clinical supervision after the discharge during 3 months
Duration of incubation period at typhoid fever?
3-7 days
9-21 day
From a few hours to 2-3 days
From 12 to 100 days
From a few hours to 17 days
Duration of incubation period at typhoid fever?
30-45 days
3-7 days
From a few hours to 2-3 days
9-21 day
From a few hours to 12 days
Entrance of causative agent at typhoid fever?
Mucous membrane of amygdales
Mucous membrane of nasopharynx
Epithelial cells of skin
Mucous membrane of colon
Mucous membrane of digestive tract
For proven of Salmonella typhus infection use:
Test of Cuverkalov
RA with O- and Н-antigen
PCR
Bacteriologic examination and reaction of Vidal
Bacteriologic examination and RNGA with a Vi-antigen
For identification of transmitter of Salmonella typhi utilize:
Bacteriologic examination and RNGA with a Vi-antigen
RIHA with Н-antigen
Bacteriologic examination and reaction of Vidal
General blood analysis
E.
29.
A. *
B.
C.
D.
E.
30.
A. *
B.
C.
D.
E.
31.
A. *
B.
C.
D.
E.
32.
A.
B.
C.
D.
E. *
33.
A.
B.
C.
D. *
E.
34.
A.
B.
C.
D. *
E.
35.
A. *
B.
C.
D.
E.
36.
A.
Test of Paula- Bunnela
Greatest epidemiological role in the salmonellosis belongs.
Ducks eggs
Gray rats
Pigs
Fish
Man
Greatest epidemiological role in the salmonellosis belongs to:
Cattle
Gray rats
Mice
Fish
Man
The main epidemiological role in the salmonella belongs to:
Cattle
Rodents
Birds
Fish
Horses
In a patient development of clostridial gastroenterocolitis is suspected. What is it necessary for
bacteriological confirmation of diagnosis?
Endo‘s medium, thermostat
Ploskirev‘s medium and blood agar
Blaurock‘s medium, thermostat
Endo‘s medium, anaerostat
Blaurock‘s media, anaerostat
In epidemic focus of typhoid fever has be done, except:
Daily thermometry
Coproculture
Reaction of Vidal
Hemoculture
Urine culture
At epidemic outbreak of typhoid fever has be done, except:
Daily thermometry
Coproculture
Reaction of Vidal
Hemoculture
Urine culture
In epidemic focus of typhoid fever has be done:
All enumerated
Coprologic culture
Reaction of Vidal
Daily thermometery
Urine culture
In order to prevent salmonellosis should be.
Disinfection
B.
C.
D. *
E.
37.
A.
B.
C.
D. *
E.
38.
A.
B.
C.
D. *
E.
39.
A. *
B.
C.
D.
E.
40.
A. *
B.
C.
D.
E.
41.
A.
B.
C. *
D.
E.
42.
A.
B.
C.
D.
E. *
43.
A.
B.
C. *
D.
E.
Vaccination
Chemoprophylaxis
Sanitary and epidemiological control over food
All these measures are not undertaken
To prevent salmonellosis should be done:
Disinfection
Vaccination
Chemoprophylaxis
Sanitary and epidemiological control over food industry
All these measures are not undertaken
In order to prevent salmonellosis should be.
Disinfection
All these measures are undertaken
Chemoprophylaxis
Sanitary and epidemiological control over food industry
All these measures are not undertaken
Medical workers at salmonellosis must to work in protective clothing:
Type I
Type II
Type III
Type IV
Type V
Name of the basic factors of pathogen of typhoid rod?
Vi-antigen and endotoxin
Exotoxin
Vi-antigen
Enzymes of pathogenicity
Endotoxin
Enumerate the basic factors of pathogen activity of typhoid bacteria?
Vi-antigen
Exotoxin
Vi-antigen and endotoxin
Enzymes of pathogenicity
Endotoxin
The basic factors of pathogen activity of typhoid bacteria are:
Endotoxin
Exotoxin
Vi-antigen
Enzymes of pathogenicity
Vi-antigen and endotoxin
Nowadays in Ukraine salmonellosis.
Not registered
Reported in the form of epidemics
Reported sporadically
Reported in the form of annual outbreaks
Reported only in endemic focus
44.
A.
B. *
C.
D.
E.
45.
A.
B.
C.
D. *
E.
46.
A.
B.
C. *
D.
E.
47.
A.
B. *
C.
D.
E.
48.
A.
B.
C.
D. *
E.
49.
A.
B.
C.
D. *
E.
50.
A.
One of methods of diagnostics of typhoid fever and paratyphoid fever is the selection of hemoculture.
For that in a fever period at 1-st week sowing of blood from a vein on bilious bulione or Rappoport
medium
0,5 ml of blood
10 ml of blood
15 ml of blood
20 ml of blood
25 ml of blood
One of methods of diagnostics of typhoid fever and paratyphoid fever is the selection of hemoculture.
For that in a fever period at 3-d week sowing of blood from a vein on bilious bulione or Rappoport
medium
5 ml of blood
10 ml of blood
15 ml of blood
20 ml of blood
25 ml of blood
One of methods of diagnostics of typhoid fever and paratyphoid fever is the selection of hemoculture.
For that in a fever period sowing of blood from a vein on bilious bullion or Rappoport‘s medium in
correlation 1:10 is made. What volume of blood must be taken on the 2nd week of disease?
5 ml of blood
10 ml of blood
15 ml of blood
20 ml of blood
25 ml of blood
What volume of blood must be taken on the 1st week of typhoid fever for sowing of blood from a
vein on bilious bulione or Rappaport‘s medium.
0,5 ml of blood
10 ml of blood
15 ml of blood
20 ml of blood
25 ml of blood
What volume of blood must be taken on the 3-d week of typhoid fever for sowing of blood from a
vein on bilious bulione or Rappaport‘s medium.
5 ml of blood
10 ml of blood
15 ml of blood
20 ml of blood
25 ml of blood
Patient K., applied to the infectious department suspecting food poisoning (maybe staphylococce).
What methods can confirm the diagnosis?
Diagnostic confirmation requires isolating staphyloccocci from the urine
Diagnostic confirmation requires isolating staphyloccocci from the stool
Diagnostic confirmation requires isolating staphyloccocci from the liquor
Diagnostic confirmation requires isolating staphyloccocci from the suspected food
All enumerated
Convalescents of typhoid fever may go out from clinic after:
Non-permanent negative bacteriologic examination of defecating
B. *
C.
D.
E.
51.
A. *
B.
C.
D.
E.
52.
A. *
B.
C.
D.
E.
53.
A. *
B.
C.
D.
E.
54.
A.
B.
C.
D. *
E.
55.
A.
B.
C.
D. *
E.
56.
A. *
B.
C.
D.
E.
57.
A.
B.
C.
21th day of normal temperature of body and 3-multiple negative bacteriologic examination of
excrement and urine
14th day of normal temperature of body and 2-multiple negative bacteriologic examination of
excrement and urine
Clinical convalescence and normalization of rectal manoscopic picture
Normalisations of rectal manoscopic picture and in default of title of antibodies in RNGA
Salmonella adapted to humans.
S. typhi
S. newport
S. cholerae-suis
S. abortus-ovis
S. gallinarum-pullorum
Salmonella is classified by:
O-antigen and H-antigen
O-antigen and J-antigen
H-antigen and Vi-antigen
O-antigen, H-antigen and Vi-antigen
O-antigen, H-antigen, Vi-antigen and HBsAg
Salmonella is classified by:
O-antigen and H-antigen
O-antigen and Vi-antigen
H-antigen and Vi-antigen
Z-antigen, H-antigen and Vi-antigen
HBsAg, O-antigen, F-antigen, Vi-antigen
Salmonella typhi contains:
Only O-antigen and Н-antigen
Only J-antigen and Vi-antigen
Only H-antigen and T-antigen
O-antigen, H-antigen and Vi-antigen
O-antigen, HCV, Vi- antigen and HBsAg
Salmonella typhus contains:
Only O-antigen and Н-antigen
Only O-antigen and Vi-antigen
Only H-antigen and Vi-antigen
O-antigen, H-antigen and Vi-antigen
O-antigen, H-antigen, Vi- antigen and HBcAg
Site of causative agent entrance at typhoid fever?
Mucous membrane of small intestine
Mucous membrane of nasopharynx
Epithelial cells
Mucous membrane of genital organs
Mucous membrane of esophagus
Site of causative agent entrance at typhoid fever?
Mucous membrane of amygdales
Mucous membrane of pharynx
Epithelial cells of skin
D.
E. *
58.
A.
B.
C. *
D.
E.
59.
A.
B.
C.
D. *
E.
60.
A.
B. *
C.
D.
E.
61.
A.
B. *
C.
D.
E.
62.
A.
B. *
C.
D.
E.
63.
A.
B. *
C.
D.
E.
64.
A.
B. *
C.
D.
Mucous membrane of colon
Mucous membrane of small intestine
The etiologic diagnosis of acute intestinal infections can be confirmed thus, except:
Separation of pathogen from patients and from food debris
To obtain identical cultures of bacteria from a few patients from those which consumed that meal
Separation of identical cultures from different materials (washings, vomiting mass, excrement) at
one patient at the bacterial semination no less than 105/g and diminishing of this index in the process
of convalescence
Presence at the selected culture of Escherichia’s and staphylococcus enterotoxin
Positive agglutination reaction or other immunological reactions with autostrains of possible
pathogen, which testify to growth of title of antibodies on the blood serum of patient in the dynamics
of
Thedisease
kind of immunity after salmonellosis.
Passive
Not formed
Short term
Type specific
Inherited
The kind of immunity after salmonellosis.
Inherited
Type specific
Short term
Not formed
Permanent
What immunity formed after salmonellosis.
Inherited
Type specific
Short term
Not formed
Passive
The part of reproduction of typhoid bacterium in the organism of human is:
Stomach
Lymphatic formations of colon
Blood
Bilious ways
Mucous membrane of colon
Place of reproduction of typhoid bacterium in the organism:
Stomach
Lymphatic formations of colon
Blood
Bilious ways
Mucous membrane of colon
The part of reproduction of typhoid bacterium in the organism of human is:
Oral cavity
Lymphatic formations of colon
Blood
Nerve ganglions
E.
65.
A.
B. *
C.
D.
E.
66.
A.
B. *
C.
D.
E.
67.
A.
B. *
C.
D.
E.
68.
A.
B. *
C.
D.
E.
69.
A.
B.
C.
D.
E. *
70.
A. *
B.
C.
D.
E.
71.
A.
B.
C.
D. *
E.
72.
A.
B.
Mucous membrane of colon
The rules of discharging from a hospital of salmonellosis patients.
One-time negative bacteriological investigation of stool
Three negative bacteriological investigation of stool
14 days normal body temperature and the double negative bacteriological study stool and urine
Clinical recovery and normalization rectomanoscopy picture
Normalization rectomanoscopy picture and in the absence of antiserum to RNGA
The rules of discharging from hospital of salmonellosis patients.
Two-time negative bacteriological investigation of stool and normalization rectomanoscopy picture
Three negative bacteriological investigation of stool
21 days normal body temperature and the double negative bacteriological study stool and urine
Normal temperature and clinical recovery
Normalization rectomanoscopy picture and in the absence of antiserum to RNGA
The source of the causative agent of salmonellosis are.
Urine of patients
Farm birds
Rodents
Soil
Feces of patients
The source of the causative agent of salmonellosis is:
Sick person
Farm animals
Rats
Environment
Stool of patients
At salmonellosis the source of the causative agent is:
Soil
Feces of patients
Rodents
Sick person
Farm animals
Typhoid bacilli are usually cultured from:
Blood, stool, urine
Blood, urine, sputum
Stool, liquor, urine
Blood, stool, sputum
Stool, liquor, sputum
Typhoid bacilli are usually cultured from:
Blood, stool, sputum
Blood, urine, sputum
Stool, liquor, urine
Blood, stool, urine, bile
Stool, liquor, sputum
Typical food factor in the transmission of salmonella is.
Acidic juices
Alcohol drinks
C. *
D.
E.
73.
A.
B.
C.
D.
E. *
74.
A.
B.
C. *
D.
E.
75.
A. *
B.
C.
D.
E.
76.
A. *
B.
C.
D.
E.
77.
A. *
B.
C.
D.
E.
78.
A. *
B.
C.
D.
E.
79.
A.
B.
C. *
D.
E.
80.
Meat products
Salad of red beets
Compote of dried fruits
In the transmission of salmonella the common food factors is:
Acidic juices
Compote of apple
Alcohol drinks
Salad of potatoes
Meat products
Typical food factor in the transmission of salmonella is.
Orange juice
Fried potatoes
Meat products
Salad of fresh cabbage
Compote of fresh fruits
What changes in general analysis of blood are characteristic for typhoid?
Leucopenia, aneosinophilia, lympho-, monocytosis, increasing of ESR
Leucocytosis, hypereosinophilia, thrombocytosis, increasing of ESR
Hypochromic anaemia, leucocytosis, appearance of young forms, ESR is not changed
Hyperchromatic anaemia, leycocytosis, appearance of young forms, increasing of ESR
Leucopenia, lymphopenia, thrombocytosis, increasing of ESR
What changes in general blood analysis are characteristic for typhoid fever?
Leucopenia, aneosinophilia, lympho-, monocytosis, increasing ESR
Leucocytosis, hypoeosinophilia, thrombocytosis, decreasing ESR
Anaemia, leucocytosis, appearance of young forms, ESR is not changed
Hyperchromic anaemia, leucocytosis, appearance of young forms, increasing ESR
Leucopenia, lymphopenia, thrombocytosis, increasing ESR
Such changes in general blood analysis are character for typhoid fever?
Leucopenia, aneosinophilia, lympho-, monocytosis, increasing ESR
Leucocytosis, hypereosinophilia, thrombocytosis, increasing ESR
Hypochromatic anaemia, leucocytosis, appearance of young forms, ESR is not changed
Hyperchromatic anaemia, leycocytosis, appearance of young forms, increasing ESR
Leucopenia, lymphopenia, thrombocytosis, increasing ESR
What does the diagnostic titre of reaction of Vі-haemaglutination testify to?
About typhoid bacterial-carrier
About the period of height of the epidemic typhus
About meningococcaemia
About a malaria
About the latent period of brucellosis
What environments do typhoid rods grow on well?
Chicken embryos
Water-serum nourishing environment
Bilious clear soup
Meat-peptone gelose + cistin
To the Bismute-sulfate gelose
What ever symptom is not characteristic for typhoid on the second week of illness?
A.
B.
C.
D.
E. *
81.
A. *
B.
C.
D.
E.
82.
A. *
B.
C.
D.
E.
83.
A. *
B.
C.
D.
E.
84.
A. *
B.
C.
D.
E.
85.
A.
B. *
C.
D.
E.
86.
A.
B.
C. *
D.
E.
87.
A.
B. *
C.
D.
Constipation
Headache
Fever
Relative bradycardia
Cramps
What from the adopted phases of pathogenesis is not characteristic for typhoid?
Swelling, edema of mucous membrane of overhead respiratory tracts
Stage of penetration
Stage of lymphodefence reactions
Stage of bacteriaemia
Stage of intoxication
What from the adopted ways of transmission is characteristic for typhoid?
Alimentary
Contact
Transmission
Air-drop
Vertical
What from the indicated pathology anatomic phases is not characteristic for typhoid?
Catarrhal inflammation of rectum
Cerebral-type of swelling
Necrosis
Ulcers
Clean ulcers
What from the listed signs is not character for typhoid rash?
Papules, disappear together with normalization of body temperature
Appears on a 7-10-th day, maculopapular (roseola-type) rash
Located mainly on an abdomen and lateral surfaces of trunk, observed at the half of patients
The amount of elements is limited
Roseola-type, sometimes saved longer than fever
What from the listed signs is not character for typhoid rash?
Papules, not disappear together with normalization of body temperature
Appears on a 2-5-th day, vesicular rash
Located mainly on an abdomen and lateral surfaces of trunk, observed at the half of patients
The amount of elements is limited
Roseola-type, sometimes saved longer than fever
What from the listed signs is not character for a typhoid rash?
Papules, not disappear together with normalization of body temperature
Appears on a 7-10-th day, maculopapular (roseola-type) rash
Located mainly on an extremities, observed at all patients
The amount of elements is limited
Roseola-type, sometimes saved longer than
What group of infectious diseases salmonellosis belong to?
Sapronosis
Zoonosis
Antroponosis
Zooantroponosis
E.
88.
A.
B.
C.
D. *
E.
89.
A.
B.
C. *
D.
E.
90.
A. *
B.
C.
D.
E.
91.
A.
B.
C.
D.
E. *
92.
A. *
B.
C.
D.
E.
93.
A.
B.
C.
D.
E. *
94.
A. *
B.
C.
D.
E.
95.
A. *
B.
The group is not defined
What group of infectious diseases Paratyphoid B belong to?
Sapronosis
Zoonosis
Antroponosis
Antropozoonosis
The group is not defined
What group of infectious diseases Paratyphoid A belong to?
Sapronosis
Zoonosis
Antroponosis
Zooantroponosis
The group is not defined
What inoculums material should be taken to discharge the toxins at Food poisoning?
Suspected food
Urine
Stool
Vomiting mass
Spinal marrow
What is the most important factor in Salmonellosis transmission.
Boil meat
Home made foods
Water
Sexual contact
Eggs
What is the most important factor in Salmonellosis transmission.
Row meat
Home made foods
Water
Sexual contact
Blood
What is the most important factor in Salmonellosis transmission.
Mushrooms
Frozen fruits
Water
Mosquitoes
Eggs
What is not character for typhoid fever from the pathogen phases?
Swelling, edema of mucous membrane of upper respiratory tracts
Stage of penetration
Stage of lymphoimmune reactions
Stage of bacteriemia
Stage of intoxication
What is the source of typhoid fever?
Sick human
Sick agricultural animals
C.
D.
E.
96.
A. *
B.
C.
D.
E.
97.
A. *
B.
C.
D.
E.
98.
A.
B.
C.
D. *
E.
99.
A.
B.
C.
D.
E. *
100.
A. *
B.
C.
D.
E.
101.
A.
B. *
C.
D.
E.
102.
A.
B. *
C.
D.
E.
Sick rodents
Soil
Birds
What mechanism of transmission is typical for salmonellosis.
Fecal-oral
Contact
Transmissive
Air-drop
All possible
The most typical way of transmission for salmonellosis.
Alimentary
Contact
Water
Air-drop
Flies
What mechanism of transmission is typical for salmonellosis.
Vertical
Contact
All possible
Fecal-oral
Transmissive
What group of infections salmonellosis belong to?
All enumerated
Parenteral
Respiratory
Blood
Intestinal
What group of infectious diseases salmonellosis belong to?
Intestinal
Blood
Respiratory
External coverings
Vertical
What percentage of chronic carrier state can form after salmonellosis.
Not formed
Formed in 0,1-1 % of the cases
Formed in 8-10 % of cases
Formed in 20-30 % of cases
Formed in 50-80 % of cases
What percentage of chronic carrier state can form after salmonellosis.
Formed in 100 % of the cases
Formed in 0,1-1 % of the cases
Formed in 5-10 % of cases
Formed in 20-30 % of cases
Not formed
103.
A. *
B.
C.
D.
E.
104.
A.
B.
C.
D. *
E.
105.
A. *
B.
C.
D.
E.
106.
A. *
B.
C.
D.
E.
107.
A. *
B.
C.
D.
E.
108.
A.
B.
C.
D.
E. *
109.
A.
B.
C.
D.
E. *
110.
A.
B. *
What preventive and antiepidemic activities against Salmonella focused on the first link of epidemic
process.
Veterinarian measures
Revealing, hospitalization and treatment of sick people
Systematic sanitary-hygienic control
Disinfection
Vaccination
What way of transmission is not inherent to Salmonella typhimurium.
Milk
Contact home
Water
Sex
Food
What Salmonella is adapted to humans?
S. typhi
S. newport
S. cholerae-suis
S. abortus-ovis
S. gallinarum-pullorum
What Salmonella is adapted to humans?
S. enteritidis
S. newport
S. cholerae-suis
S. abortus-ovis
S. gallinarum-pullorum
What Salmonella is adapted to humans?
S. typhimurium
S. newport
S. cholerae-suis
S. abortus-ovis
S. gallinarum-pullorum
What symptom is not characteristic for typhoid fever on the second week of disease?
Constipation
Headache
Fever
Relative bradycardia
Cramps
What symptom is not characteristic for typhoid fever on the second week of disease?
Hepatosplenomegalia
Headache
Fever
Rash
Cramps
What term of looking after the source of typhoid fever?
14 days
21 days
C.
D.
E.
111.
A.
B.
C.
D.
E. *
112.
A. *
B.
C.
D.
E.
113.
A.
B.
C.
D. *
E.
114.
A.
B.
C.
D. *
E.
115.
A. *
B.
C.
D.
E.
116.
A.
B.
C.
D.
E. *
117.
A.
B.
C.
D.
E. *
118.
7 days
30 days
Not needed
What term of looking after the source of typhoid fever?
52 days
Not needed
71 days
30 days
21 days
What term of looking after the source of typhoid fever?
21 days
3-6 days
14 days
45 days
90 days
What way of transmission is not inherent to Salmonella typhy.
Milk
Contact home
Water
Sex
Food
What way of transmission is not inherent to Salmonella typhy.
Milk
Contact home
Water
Transmissive
Food
What ways of transmission is the most character for typhoid fever?
Alimentary
Contact
Transmissive
Air-drop
Vertical
When is it possible to abolish etiotropic drugs in a patient with typhoid fever?
After normalization of body temperature
After normalization of sizes of liver and spleen
After disappearance of roseollas
In 10 days after disappearance of roseollas
After the 10th day of normal body temperature
When is it possible to abolish etiotropic drugs in a patient with typhoid fever?
Direct after normalization of body temperature
After normalization of liver and spleen sizes
After appearance of roseollas
In 10 days after positive serologic reactions
After the 10th day of normal body temperature
When is it possible to abolish etiotropic preparations in a patient with typhoid fever?
A.
B.
C.
D.
E. *
119.
A.
B.
C. *
D.
E.
120.
A.
B.
C. *
D.
E.
121.
A.
B.
C. *
D.
E.
122.
A.
B.
C. *
D.
E.
123.
A.
B.
C. *
D.
E.
124.
A.
B.
C. *
D.
E.
125.
A.
B. *
C.
D.
Direct after normalization of body temperature
After normalization of sizes of liver and spleen
After disappearance of roseollas
In 10 days after disappearance of roseollas
After the 10th day of normal temperature of body
When there can be such specific complication of typhoid fever, like to intestinal bleeding?
In the incubation period
On the 4th week of illness
On the 3d week of illness
After stopping the antibiotic therapy
On the any week of illness
When there can be such specific complication of typhoid fever, like to perforation of bowel?
On the 1st week of illness
On the 2nd week of illness
On the 3rd week of illness
On the 4th week of illness
On the any week of illness
When there can be such specific complication of typhoid fever, like intestinal bleeding?
On the 1st week of illness
On the 2nd week of illness
On the 3rd week of illness
On the 4th week of illness
On the any week of illness
When there can be such specific complication of typhoid fever, like to perforation of bowel?
In the incubation period
On the 4th week of illness
On the 3d week of illness
After stopping the antibiotic therapy
On the any week of illness
When there can be such specific complication of typhoid fever, like intestinal bleeding?
In the latent period
On the 5th week of illness
On the 3d week of illness
Cannot develop
On the any week of illness
When there can be such specific complication of typhoid fever, like the perforation of bowel?
In the latent period
On the 5th week of illness
On the 3d week of illness
Cannot develop
On the any week of illness
Which from the listed products can become the causal factor of toxic food-borne infection?
Decorative cakes
Galantine
Cheese
Fresh bread
E.
126.
A.
B.
C.
D.
E. *
127.
A.
B.
C.
D.
E. *
128.
A.
B.
C.
D.
E. *
129.
A.
B.
C.
D. *
E.
130.
A.
B.
C.
D.
E. *
131.
A. *
B.
C.
D.
E.
132.
A. *
B.
C.
D.
E.
133.
A.
B.
Tea
Which season is prevalence for salmonellosis.
Spring
Winter and spring
Autumn
Winter
Summer-autumn
Which season is prevalence for salmonellosis.
All the year round
Exactly August
Autumn
Winter
Summer-autumn
Which season is prevalence for salmonellosis.
Spring-summer
Only spring months
Autumn
Winter
Summer-autumn
Which type of outbreaks in salmonellosis is the main?
Water
Home
Farm
Food
Milk
Which type of outbreaks in salmonellosis is the main?
Contact
Blood
Wound
Milk
Food
Which type of outbreaks in salmonellosis is the main?
Food
Contact
Sexual
Nosocomial
Water
Who is the source of typhoid fever?
Sick person
Sick livestock animals
Sick rodents
Soil
Reptiles
A child is diagnosed with giardiasis. What preparation is it more expedient to apply for treatment?
Ursohol
Delagil
C. *
D.
E.
134.
A.
B.
C. *
D.
E.
135.
A.
B.
C.
D. *
E.
136.
A.
B.
C.
D.
E. *
137.
A. *
B.
C.
D.
E.
138.
A. *
B.
C.
D.
E.
139.
A.
B. *
C.
D.
E.
140.
A.
B. *
C.
D.
E.
Ornidazol
Tetracyclin
Enterosgel
The main epidemiologic role at shigellosis play:
Sick with an acute form illnesses
Sick with a chronic form illnesses
Sick with the latent form illnesses
Healthy transmitters
Children
A patient complaints of severe abdominal pain, smelly watery diarrhea with secretion of blood. What
kind of disease might be considered previously.
Amoebiasis
Rotaviral gastroenteritis
Giardiasis
Balantidiasis
Cholera
During acute intestinal amoebiosis in feces will be:
Tissue form and cyst
Small vegetative form, pre-cystic
Small vegetative and cystic shape
Tissue and pre cystic forms
Large vegetative form
Amount of solutions necessary for the primary rehydration at cholera is.
Accordingly to the degree of dehydration at time of hospitalization
In accordance with the loss of liquid
2l
5l
10 l
Amount of solutions necessary for the primary rehydration in cholera is:
Accordingly to the degree of dehydration at time of hospitalization
In accordance with the loss of weight
3l
4l
2l
Amount of solutions necessary for the secondary rehydration in cholera is.
Accordingly to the degree of dehydration at the time of hospitalization
In accordance with the loss of liquid
2l
5l
10 l
Amount of solutions necessary for the secondary rehydration in cholera is:
Accordingly to the time of hospitalization
In accordance with the loss of liquid
3l
Every 2 hours 2 l
2l
141.
A. *
B.
C.
D.
E.
142.
A.
B.
C.
D.
E. *
143.
A.
B.
C.
D.
E. *
144.
A. *
B.
C.
D.
E.
145.
A. *
B.
C.
D.
E.
146.
A.
B. *
C.
D.
E.
147.
A. *
B.
C.
D.
E.
148.
A.
B.
C. *
At a child is diagnosed Lambliosis. What drug is the most helpful in such case?
Ornidazol
Dimedrol
Ursophalk
Quinolones
Gastrolit
At I degree of dehydration the loss of liquid is:
0,1-1,0 % of body weight
2-7 % of body weight
9-11 % of body weight
6-8 % of body weight
1-3 % of body weight
At I degree of dehydration the loss of liquid is:
0,5-1,5 % of body weight
6-9 % of body weight
3-6 % of body weight
5-8 % of body weight
1-3 % of body weight
At intestinal аmebiasis, area of mucous membrane of bowel between ulcers:
Not changed
Sinusoid without edema
Hyperemic, fillings out
Ordinary color, fillings out
Hyperemic without edema
At intestinal аmebiasis, area of mucous membrane of bowel between ulcers:
Not changed
Grey color with edema
Hyperemic, fillings in
Brown color, fillings out
Hyperemic with edema
What “metabolic violation” will be at II degree of dehydration:
Subcompensated
Negative
Irreversible
Moderate metabolic acidosis
Insignificant metabolic alkalosis
At what percent of fluid loss will be II degree of dehydration?
3-6 % of body weight
6-9 % of body weight
1-3 % of body weight
0,5-2 % of body weight
2-7 % of body weight
At what percent of fluid loss will be III degree of dehydration?
3-6 % of body weight
Over 10 % of body weight
6-9 % of body weight
D.
E.
149.
A. *
B.
C.
D.
E.
150.
A.
B.
C.
D. *
E.
151.
A.
B.
C.
D. *
E.
152.
A.
B. *
C.
D.
E.
153.
A.
B.
C.
D. *
E.
154.
A.
B. *
C.
D.
E.
155.
A.
B. *
C.
D.
E.
156.
A.
4-8 % of body weight
10-15 % of body weight
At what percent of fluid loss will be the II degree of dehydration?
3-6 % of body weight
Over 10 % of body weight
6-9 % of body weight
4-8 % of body weight
10-15 % of body weight
At what percentage of fluid loss will be IV degree of dehydration?
4-8 % of body weight
6-9 % of body weight
3-6 % of body weight
Over 10 % of body weight
Over 15 % of body weight
At what percentage of fluid loss will be the IV degree of dehydration?
1-3 % of body weight
0,1-1 % of body weight
8-9 % of body weight
Over 10 % of body weight
Over 15 % of body weight
Choose the universal drug given for amoebiasis from the list below.
Osarsol
Metronidazole
Tetracycline
Delagil
All are correct
Drug of choice for sanation of the holders of amoeba cysts can be.
Monomitsin
Delagil
Tetracycline
Furamid
Ursosan
Duration of therapy of primary rehydration in cholera is.
30 minutes
2 hours
6 hours
12 hours
1 days
Duration of therapy of primary rehydration in cholera is:
1 hour
2 hours
12 hours
9 hours
1 day
Endotoxin is not contained by shigella:
Boyd
B.
C.
D.
E. *
157.
A.
B.
C.
D.
E. *
158.
A.
B. *
C.
D.
E.
159.
A. *
B.
C.
D.
E.
160.
A.
B.
C.
D. *
E.
161.
A.
B.
C.
D. *
E.
162.
A.
B.
C.
D. *
E.
163.
A.
Grigor'ev-Shig
Zonne
Fleksner
All enumerated contain
Etiologic agent of dysentery is:
Sh. dysentery
Sh. zonnei
Sh. flexneri
Sh. boydii
All enumerated
Etiology agent of Shigellosis is:
Sh. dysentery
All enumerated
Sh. flexneri
Sh. boydii
Sh. zonnei
For specific therapy used for ameobiasis that adversely acts on vegetative and tissue forms of
amoeba. Which group does delagil belong to:
Tissue anti amoeboids
Indirect anti amoeboids
Direct anti amoeboids
Products with universal effect
Do not belong to any of these groups
For specific therapy used for ameobiasis that adversely acts on vegetative and tissue forms of
amoeba. Which group does metronidasol belong to:
Tissue antiamoeboids
Indirect antiamoeboids
Direct antiamoeboids
Drugss with universal effect
Do not belong to any of these groups
For the rehydration at dehydration shock it is necessary to conduct the permanent careful account of
all losses of liquid every:
4 hrs
30 min
3 hrs
2 hrs
10 min
For the rehydration in dehydration shock it is necessary to conduct the permanent careful account of
all losses of liquid in each:
1 hr
30 min
4 hrs
2 hrs
5 hrs
For verification of diagnosis of balantidiasis more frequently used test is:
Virological researches
B.
C.
D. *
E.
164.
A.
B.
C. *
D.
E.
165.
A.
B.
C. *
D.
E.
166.
A.
B.
C.
D. *
E.
167.
A.
B.
C.
D. *
E.
168.
A. *
B.
C.
D.
E.
169.
A.
B.
C.
D. *
E.
170.
A.
B. *
C.
D.
Bacteriological examinations
Roentgenologic researches
Research on protozoa
Ultrasound investigation
How long does last health system control of convalescence after balantidiasis?
6 months
3 months
1 year
2 years
5 years
How long is continuing clinical convalescence after acute amoebiasis.
3-6 months
1-3 months
6-12 months
12-18 months
18-24 months
How long the incubation period last, during dysentery:
1-2 hours
2-3 days
10-15 hours
5-7 days
7-10 days
How long the incubation period last during dysentery:
1-2 days
3-4 days
12-24 hours
5-7 days
14-21 days
How often treatment of cyst amebae carrier should be done?
Twice on a year
Three times on a year
One time on a year
Does not treat
Quarterly
How to increase frequency of findings of lamblia cyst in fresh feces and vegetative forms in duodenal
content?
Cultivation in thermostat
By the method of floatation in bilious clear soup
Cultivation in anaerobic chamber
By the applications of phase-contrast and lumencense microscopy with the help of methylen-orange
To cultivate on a nourishing environment
Immunity after carried shigellosis:
Tense and species-specific
Untense and type specific
Lifelong and cross
Untense and cross
E.
171.
A.
B. *
C.
D.
E.
172.
A.
B.
C. *
D.
E.
173.
A.
B.
C. *
D.
E.
174.
A.
B.
C. *
D.
E.
175.
A.
B.
C. *
D.
E.
176.
A.
B.
C.
D. *
E.
177.
A. *
B.
C.
D.
E.
178.
Not formed
Immunity after shigellosis is:
Innate immunity
Untense and type specific
Cross and permanent
Formed only after vaccination
Not formed
In 1 liter of Trisalt solution, the concentration of potassium chloride is:
3 g/l
1.5 g/l
1.0 g/l
2 g/l
2.5 g/l
In 1 liter of Trisalt solution, the concentration of sodium bicarbonate is:
3 g/l
15 g/l
40 g/l
2 g/l
25 g/l
In a different places of village found out a few cases of cholera. Who in the focus of cholera was send
in an insulator?
Carriers
Persons contact with the patient
Patients with cholera
Persons with dysfunction of alimentary tract
Persons with hyperthermia
In a different places of settlement found out a few cases of cholera. Who in the focus of cholera was
send in an infectious hospital?
Carriers
Persons that were in contact with the patient
Patients with cholera
Persons with dysfunction of alimentary tract
Persons with hyperthermia
In a settlement was found out a few cases of cholera. Who must be insulated?
Persons with disfunction of intestine
Patients with cholera
Carriers
Persons, were in contact with the cholera patient
Persons with hyperthermia
In acute intestinal amoebiasis causative agent is found in stool in which form:
Tissue forms
Minor vegetative form
Vegetative form
Cysts
All listed forms
In the outbreak of cholera it is necessary to carry out such measures, except:
A.
B.
C.
D.
E. *
179.
A.
B.
C. *
D.
E.
180.
A.
B.
C.
D.
E. *
181.
A.
B.
C. *
D.
E.
182.
A.
B.
C.
D.
E. *
183.
A. *
B.
C.
D.
E.
184.
A. *
B.
C.
D.
E.
185.
Hyperchlorination of drinking water
An active discovery of patients by rounds
Obligatory hospitalization, inspection and treatment of patients and vibrio tests
Revealing and isolation of contact persons
Vaccine prophylaxis
In the different places of settlement found out a few cases of cholera. Who from the contacts of
cholera patient is sent in an isolation ward?
Vibrio positive
Patients with cholera
Contact with the patient persons
Persons with dysfunction of intestine
Persons with high temperature
In the different places of settlement found out a few cases of cholera. Who from the contacts of
cholera patient is sent in an isolation ward?
Vibriocarriers
Persons that were in contact with the patient
Persons with dysfunction of alimentary tract
Persons with hyperthermia positive
Contact with the patient persons
In the different places of settlement it is found out a few cases of cholera. Who from such place is
directed to an isolation ward?
Patients with a cholera
Transmitters
Persons who had contact with the patient
Persons with dysfunction of gastro-intestinal tract
Persons who left the place on infection
In the outbreak of cholera it is necessary to carry out such measures, except:
Hyperchlorination of drinking water
An active search of patients by rounds
Obligatory hospitalization, inspection and treatment of patients and vibrio tests
Revealing and isolation of contact persons
Antiviral treatment
In what disease during stool analysis mucous, erythrocytes, eosinophils, plasma cells and crystals
Charcot-Leiden were reveals.
Intestinal amoebiasis
Intestinal yersiniosis
Shigellosis
Balantidiasis
Enterohaemorrhagic esherichiosis
In what disease during stool analysis mucous, erythrocytes, eosinophils, plasma cells and crystals
Charcot-Leiden were reveals?
Intestinal аmoebiasis
Intestinal Yersiniosis
Salmonellosis
Giardiasis
Rotaviral infection
In which form during acute intestinal amoebiasis causative agent is found in stool:
A. *
B.
C.
D.
E.
186.
A.
B.
C.
D. *
E.
187.
A.
B. *
C.
D.
E.
188.
A.
B.
C. *
D.
E.
189.
A.
B.
C. *
D.
E.
190.
A. *
B.
C.
D.
E.
191.
A.
B. *
C.
D.
E.
192.
A. *
B.
C.
Tissue forms
Small vegetative form
Nothing
Cysts
All listed forms
Intestinal amoebiasis can be characterized by such complications, ecxept:
Amoeboma
Intestinal bleeding
Perforation of ulcers
Meningitis
Stricture of colon
Intestinal amoebiasis can be characterized by such complications, ecxept:
Liver abscess
Osteomyelitis
Perforation of ulcers
Lungs abscess
Stricture of colon
Method of etiotropic therapy of cholera is.
Glucocorticoids
Antiviral
Antibiotics
Rehydration
Vaccine
Method of etiotropic therapy of cholera is.
Non steroid anti inflammatory
Antifungal
Antibiotics
Desintoxication
Vaccine
Most of vegitative forms of amoebiasis can be found in stool of:
The patient with acute intestinal amoebiasis
Convalescent after acute intestinal amoebiasis
Defecation after use laxative
Patients with chronic recurrent intestinal amoebiasis in remission stage
Patients with amebic liver abscesses
Mother with a child hospitalized with the diagnosis of shigellosis, typical form, moderate severity.
Other family members are healthy. What measures are conducted to contact persons?
Supervision 7 days. General blood analyses
Supervision 7 days. Non-permanent bacteriological investigation of feces on a dysenteric group
Supervision 2 weeks. Non-permanent bacteriological inspection of feces on a dysenteric group
Supervision during 24 hours from the moment of detection of patient
Hospitalization of contact persons on 7 day. Non-permanent blood analysis
Name the cause of amoebiasis.
E.histolytica
L.canicola
S.derby
D.
E.
193.
A. *
B.
C.
D.
E.
194.
A. *
B.
C.
D.
E.
195.
A. *
B.
C.
D.
E.
196.
A.
B.
C.
D.
E. *
197.
A.
B.
C.
D.
E. *
198.
A.
B. *
C.
D.
E.
199.
A.
B. *
S.boydii
B.enterocolitica
Name the cause of amoebiasis.
E.histolytica
L.icterohemorrhagica
S.sonnei
E.coli
B.anthracis
On еndoscopical inspection of a patient are found cysts of аmoeba. These changes are specific for
what disease:
Chronic intestinal аmoebiosis
Acute intestinal аmoebiosis
Amoeba liver abscess
Heterospecific ulcerous colitis
Cancer of rectum
On еndoscopical inspection of a patient are found cysts of аmoeba. For what disease are these
changes specific?
Chronic intestinal аmoebiosis
Convalescent after acute intestinal аmoebiosis
Nonspecific ulcerous colitis
Proctosigmoiditis
Cancer of rectum
Patient with cholera has bradycardia, low blood pressure, weakness. What is most important factor in
the given clinical situation?
Hypocalcemia
Hypokaliemia
Hyponatremia
Hypernatremia
Hyperkaliemia
Patient with cholera has bradycardia, low blood pressure, weakness. What is the most important
factor in the given clinical situation?
Hypercalcemia
Hypokaliemia
Hyponatremia
Hypercalcemia
Hyperpotassemia
Patient with cholera on a background treatment has signs of hyperkalemia. What solution must be
applied for further rehydration therapy?
Neohemodez
Disalt
Chlosalt
Polyhydrid
Lactasalt
Patient with cholera on a background treatment has signs of hyperkalemia. What solution must be
applied for further rehydration therapy?
Reosorbilact
Disalt
C.
D.
E.
200.
A.
B.
C.
D. *
E.
201.
A.
B.
C.
D.
E. *
202.
A.
B.
C.
D.
E. *
203.
A.
B.
C.
D.
E. *
204.
A. *
B.
C.
D.
E.
205.
A.
B.
C.
D. *
E.
206.
A.
B.
C.
D.
E. *
207.
Trisalt
Isotonic solution
Glucose 5%
Drugs of choice for the treatment of amoebae cyst carrier:
Monomycinum
Delagilum
Tetracyclin
Furamid
Ursosan
Principles of treatment of patients with shigellosis.
Diet
Antibacterial drugs
Correction of dysbacteriosis
Detoxication therapy
All enumerated
Drugs of choice at treatment of patients with shigellosis.
Probiotics
Antibacterial medicines
Prebiotics
Sorbents
All enumerated
Shigella that does not contain endotoxin:
Boidii
Grigor'eva-Shiga
Zonne
Fleksneri
All of mentioned contain
Such concomitant diseases are importent in more protracted convalescent transmitter of shigella:
HIV-infection/AIDS
Chronic hepatitis
Chronic pancreatitis
Adenoviral infection
Diabetes mellitus
The balantidiasis possible complications are all, except:
Hypochromic anaemia
Enterorrhagia
Perforation of ulcer
Abscess of liver
Cachexia
The causal agent of amoebiasis during life cycle can be in any form except:
Mature cyst
Immature cyst
Trophozoite
Quadrinucleate cyst
Spore
The causative agent of amoebiasis during life cycle can be in any form except:
A.
B.
C.
D.
E. *
208.
A. *
B.
C.
D.
E.
209.
A.
B.
C.
D. *
E.
210.
A.
B.
C.
D.
E. *
211.
A.
B.
C.
D.
E. *
212.
A.
B.
C.
D. *
E.
213.
A.
B.
C.
D.
E. *
214.
A. *
B.
C.
Mature cyst
Immature cyst
Trophosoit
Quadrinucleate cyst
Spore
The clinical forms of balatidiasis are all, except
Flash rapid
Acute
Subclinical
Chronic continues
Chronic recurrent
The drug of choice for treatment of the carriers of amoeba cysts can be.
Doxicyclin
Streptomycin
Furasolidon
Furamid
Fluconasol
The essential therapy for cholera is:
Diet
Antibacterial drugs
Correction of dysbacteriosis
Desintoxication
Primary rehydration
The essential therapy for cholera is.
Regime
Secondary rehydration
Correction of dysbacteriosis
Antiviral treatment
Primary rehydration
The inhabitant of a village, admitted with complaints about severe pain in a stomach, and diarrhea.
The stool is liquid with mixtures of blood. About what disease it is possible to think?
Amebiasis
Rotavirus gastroenteritis
Giardiasis
Shigellosis
Cholera
The main principles of therapy for rehydration in cholera is.
Determining the definitive degree of dehydration from clinical data
Amount of lost liquid which was preceded at time of hospitalization
Application of isotonic solution
Simultaneous introduction of liquid in more than one vessel
All are correct
The majority of vegetative forms of Entamoeba can be found in stool from:
The patients with acute intestinal amoebiasis
The convalescent after acute intestinal amoebiasis
Patients with amebic liver abscesses after using laxative
D.
E.
215.
A.
B.
C. *
D.
E.
216.
A. *
B.
C.
D.
E.
217.
A. *
B.
C.
D.
E.
218.
A. *
B.
C.
D.
E.
219.
A. *
B.
C.
D.
E.
220.
A.
B.
C.
D. *
E.
221.
A.
B.
C.
D. *
E.
222.
A.
Patients with chronic recurrent intestinal amoebiasis in remission stage
Patients with amebic liver abscesses
The most important epidemiologic role at shigellosis play:
Sick persons with an acute form of illnesses
Sick persons with a chronic form of illnesses
Sick with the latent form of illnesses
Healthy carriers
Children
The source of agent at shigellosis is:
Patient
Sick cattle
Sick rodents
Soil
Feces of patients
The source of exciter at shigellosis is:
Sick man
Sick agricultural animals
Sick birdss
Environment
Urine of patients
What agent can cause balantidiasis.
B. Coli
B. Enterocolitica
S. Derby
S. Boydi
L. Canicola
What agent causes balantidiasis.
B. Coli
E. Coli
Sh. Sonne
S. Enteritidis
B. Anthracis
What are the clinical forms of amoebiasis, except:
Enteric
Skin
Liver abscess
Myocarditis
Lung abscess
What are the known clinical forms of amoebiasis:
Enteric
Skin
Liver abscess
All enumerated
Lung abscess
What are the stages of life-cycle of balantidium.
Cyst
B.
C. *
D.
E.
223.
A. *
B.
C.
D.
E.
224.
A.
B.
C.
D.
E. *
225.
A.
B.
C. *
D.
E.
226.
A.
B.
C. *
D.
E.
227.
A.
B.
C.
D.
E. *
228.
A.
B.
C.
D. *
E.
229.
A.
B.
C.
D. *
E.
Vegetative and spore
Vegetative and cyst
Spore
Vegetative
What clinical forms of balantidiasis are the most often.
Mild
Acute
Subclinical
Chronic
Fulminant
What complication is typical for balantidiasis.
Intestinal bleeding
Cachexia
Perforated ulcer
Abscess of bowel
All enumerated
What does belong to the group of pathogens of amoebiasis?
Mycoplasma
Fungi
The simplest
Rickettsiae
Worms
What does belongs to the group of pathogens of amoebiasis?
Viruses
Chlamydia
The simplest
Fungi
Parasites
What from the given measures is made during the secondary rehydration?
Determining degree of dehydration from clinical data
Amount of lost liquid, which was preceded at the time of hospitalization
Application of isotonic crystalloid solutions
Simultaneous introduction of liquid in a few vessels
Amount of liquid loss
What from the below is a complication of cholera?
Collapse
Infectious-toxic shock
Acute renal insufficiency
Dehydration shock
Status typhosus
What from the below mentioned drugs can be used for the treatment of primary rehydration?
Lactosalt
Disalt
Acesalt
Trisalt
Chlosalt
230.
A.
B. *
C.
D.
E.
231.
A. *
B.
C.
D.
E.
232.
A.
B.
C. *
D.
E.
233.
A. *
B.
C.
D.
E.
234.
A.
B.
C.
D.
E. *
235.
A. *
B.
C.
D.
E.
236.
A.
B.
C. *
D.
E.
237.
A. *
B.
C.
What from the below mentioned preparations cannot be used for the treatment of primary
rehydration?
Lactosalt
Disalt
Acesalt
Trisalt
Chlosalt
What from the below mentioned preparations, can be used for the treatment of primary rehydration?
Quartasalt
Acesalt
Chlosalt
Disalt
Lactosalt
What from the below mentioned preparations, cannot be used for the treatment of primary
rehydration?
Acesalt
Trisalt
Cryoplasma
Chlosalt
Lactosalt
What from the below mentioned preparations, can be used for the treatment of primary rehydration?
Rehidron
Acesalt
Chlosalt
Quartasalt
Lactosalt
What from the given preparations can be applied for etiotropic therapy of ameobiasis?
Osarsol
Metronidazol
Tetracycline
Delagil
All are correct
What group of infectious diseases balantidiasis belongs to.
Intestinal
Blood infection
Sapronosis
External coverings
Respiratory
What group of pathogens balantidiasis belong to.
Fungi
Viral
Simplest
Parasites
Rickettsiosis
What is a source of the causal agent of amoebiasis?
People
Cows
Sheep
D.
E.
238.
A. *
B.
C.
D.
E.
239.
A.
B.
C.
D. *
E.
240.
A. *
B.
C.
D.
E.
241.
A. *
B.
C.
D.
E.
242.
A. *
B.
C.
D.
E.
243.
A. *
B.
C.
D.
E.
244.
A.
B.
C. *
D.
E.
Pigs
Camel
What is an incubation period for intestinal amoebiasis:
From 1 week to several months
3-5 days
4-6 days
1-2 years
From 3 months to 1 year
What is the incubation period for balantidiasis.
7-14 days
5-10 days
1-3 months
1-3 weeks
3-6 weeks
What is the main method of taking of material for parasitological examination in case of intestinal
amoebiasis.
Immediately after the defecation
In the next day of defecation
After processing of disinfectants
1-2 hours after processing with Lugol solution
2-3 hours after processing with iron hematoxylin
What is the main method of taking of material for parasitological examination in case of intestinal
amoebiasis.
Immediately after the defecation warm feces
In the next day of defecation
After processing of disinfectants
After using the antibiotics
2-3 hours after defecation
What is the mechanism of transmission at amoebiasis
Fecal-oral
Vector borne
Air-dropping
Wound
Vertical
What is the way of transmission of amoebiasis
Fecal-oral
Transmissive
Air-droplet
Parenteral
By flies
What is the pathological changes in intestine at balantidiasis.
No changes
Ulcer
Hyperemia, edema
Edema
Hyperemia without edema
245.
A. *
B.
C.
D.
E.
246.
A.
B. *
C.
D.
E.
247.
A.
B.
C.
D.
E. *
248.
A. *
B.
C.
D.
E.
249.
A.
B.
C.
D.
E. *
250.
A. *
B.
C.
D.
E.
251.
A.
B.
C.
D.
E. *
252.
A. *
What is the source of the causative agent of amoebiasis?
Humans
Cattle
Birds
Horses
Camels
What is the way of transmission at balantidiasis.
By mosquitoes
Food-born
Air-drop
Parenteral
Vertical
What kind of colon mucous membrane can be found between amoebiatic ulcers:
Hyperemia without edema
Lividity, without edema
Hyperemia, edema
Regular colored edema
Without changes
What kind of ulcers are present at аmebiasis?
Fillings out sharp edges, surrounded by the area of hyperemia, are placed on the unchanged mucus
membrane
Smooth sharp edges, placed on a hyperemic mucus membrane
Plain edges, placed on a hyperemic mucus membrane
Fillings out sharp edges, placed on the unchanged mucus membrane
Fillings out sharp edges, surrounded by the area of hyperemia, are placed on the changed mucus
membrane
What mechanism of shigella transmission?
Vertical
Transmissive
Air-droplet
Contact
Fecal-oral
What way of transmission at shigellosis?
Fecal-oral
Transmissive
Air-droplet
Parenteral
By flies
What part as a rule of lower GI tract is affected during amoebiasis, exept?
The descending colon
Sigmoid and rectum
The ascending colon
Transversal colon
Small intestine
What part of lower GI tract is affected with amoebiasis most often?
The descending and ascending colon
B.
C.
D.
E.
253.
A.
B.
C. *
D.
E.
254.
A.
B.
C. *
D.
E.
255.
A.
B.
C.
D. *
E.
256.
A.
B.
C.
D. *
E.
257.
A. *
B.
C.
D.
E.
258.
A. *
B.
C.
D.
E.
259.
A. *
B.
C.
Sigmoid and rectum
Duodenum and jejunum
Transversal colon
Small intestine
What solutions must be applied for compensatory rehydration in cholera?
Colloid
Hypertensive epitonic polyionic crystalloid
Isotonic crystalloid
Reosorbilact
Isotonic solution of glucose
What solutions must be applied for compensatory rehydration in cholera?
Colloid
Hypertonic polyionic crystalloid
Isotonic polyionic crystalloid
Reosorbilact
Isotonic solution of glucose
What time is it necessary to complete primary rehydration at dehydration shock?
3-5 hrs
0.5 hrs
15-20 min
1.5-2 hrs
4-6 hrs
What time is it necessary to complete primary rehydration at dehydration shock?
10-15 hrs
1/2 hrs
4-5 hrs
1.5-2 hrs
1 days
When is it possible to discharge convalescent after аmoebiasis from the hospital?
After clinical convalescence and negative results of parasitological research of excrements
After clinical convalescence and three negative results of parasitological research of excrements
After clinical convalescence and two negative results of parasitological research of excrements
After clinical convalescence and normalization of indexes of general blood analysis
After clinical convalescence, normalization of indexes of general blood analysis and two negative
results of bacteriological examination of excrement
When would you discharge a patient from hospital, who was diagnosed with balandiasis?
After clinical convalescence and two negative results of research on protozoan of excrement
After clinical convalescence and two negative results of bacteriological examination of excrement
After clinical convalescence and one negative result of parasitological research of excrement
After clinical convalescence and normalization of indexes of general blood analysis
After clinical convalescence, normalization of indexes of general blood analysis and two negative
results of bacteriological examination of excrement
Which group has pathogenic agent of giardiasis belongs to
The simplest
Worms
Rickettsiae
D.
E.
260.
A.
B. *
C.
D.
E.
261.
A.
B.
C. *
D.
E.
262.
A.
B.
C. *
D.
E.
263.
A. *
B.
C.
D.
E.
264.
A. *
B.
C.
D.
E.
265.
A. *
B.
C.
D.
E.
266.
A.
B.
C.
D.
E. *
267.
A.
Mycoplasmas
Chlamidia
Which of antibiotics are used as etiological treatment of shigellosis:
Penicillin, bicillin
Furasolidon, nifuroxasid
Tetracycline
Aminoglycosides (kanamicin)
Cephalosporins (cefazolin)
Which salt solutions do not contain potassium?
Trisalt
Lactosalt
Disalt
Quartasalt
Chlosalt
Which salt solutions do not contain potassium?
Trisalt
Lactosalt
Disalt
Quartasalt
Chlosalt
Which ulcers are specific for amoebiasis:
Purulent ulcers with undermining, surrounded by hyperemic zone located on the intact mucosa
Smooth ulcers with undermining, located on the hyperemic mucosa
Necrotic ulcers, located on the hyperemic mucosa
Edematose ulcers with undermining located on the intact mucosa
Small lesions on the basis of infiltration covered with white coat
Who is the reservoir of the causative agent at balantidiasis.
Pig
Cow
Sheep
Goat
Bear
Who is the reservoir of the causative agent in balantidiasis.
Pig
Chicken
Dog
Fox
Human
Who must be admitted in the hospital from the focus of cholera?
Carriers
Patients with cholera
Persons with dysfunction of intestine
Contact persons
All enumerated
Who must be admitted in the hospital from the focus of cholera, except?
Carriers
B.
C.
D.
E. *
268.
A.
B.
C. *
D.
E.
269.
A.
B.
C. *
D.
E.
270.
A.
B.
C.
D. *
E.
271.
A.
B.
C.
D.
E.
272.
A. *
B.
C.
D.
E.
273.
A.
B.
C. *
D.
E.
274.
A.
B.
Patients with cholera
Persons with dysfunction of intestine
Contact persons
Persons with high temperature
To the patient with the diagnosis of shigellosis antibacterial therapy is prescribed by the protracted
course. What is the most frequent complication can arise up at such treatment?
Infectious-toxic shock
Allergic reactions
Dysbacteriosis
Renal insufficiency
Toxic hepatitis
To the patient with the diagnosis of shigellosis antibacterial therapy is prescribed by the protracted
course. What is the most frequent complication can arise up at such treatment?
Disseminated intravascular coagulopathy
Allergic dermatitis
Disbacteriosis
Herpetic stomatitis
Gastric ulcer
Young farmer was diagnosed with balantidiasis. Drugs of choice would be all, except:
Monomycin
Ampicillin
Aminarson
Gentamicin
Metronidazole
Young farmer was diagnosed with balantidiasis. Drugs of choice would be:
Monomycin
Ampicillin
Metronidazole
Osarson
All enumerated
Young farmer was diagnosed with balantidiasis. Drugs of choice would be all, except:
Bisoprolol
Monomycin
Ampicillin
Metronidazole
Osarson
A Sick 18 years, hospitalized in an infectious department with diagnosis of cholera, very severe state,
dehydration of IV degree. What measures are possible primarily?
Oral rehydration by glucose solutions
Tetracycline
Intravenous stream introduction of salt solutions
Proceeding the normal microflora of intestine
Intravenous stream introduction of sodium chloride solution
All of the following are the blood flukes except:
Schistosoma japonicum
Fasciola gigantica
C.
D.
E. *
275.
A.
B.
C. *
D.
E.
276.
A.
B.
C.
D.
E. *
277.
A. *
B.
C.
D.
E.
278.
A.
B.
C.
D.
E. *
279.
A.
B.
C.
D.
E. *
280.
A.
B.
C.
D.
E. *
281.
A. *
B.
C.
D.
E.
282.
Clonorchis sinensis
Fasciola hepatica
Echinococcus granulosis
Alveococcosis belongs to:
Nematodosis
Trematodosis
Cestodosis
Ricketsiosis
Mycosis
Alveococcosis is:
Bacterial infection
Viral infection
Protozoosis
Fungal infection
Helminthosis
Ascaridiosis belongs to:
Nematodosis
Trematodosis
Cestodosis
Ricketsiosis
Mycosis
Ascaridiosis is:
Bacterial infection
Viral infection
Protozoosis
Fungal infection
Helminthosis
At I degree of dehydration the loss of liquid is:
0,5-1,5 % of body weight
6-9 % of body weight
3-6 % of body weight
5-8 % of body weight
1-3 % of body weight
At intestinal аmebiasis, area of mucous membrane of bowel between ulcers:
Hyperemic without edema
Sinusoid without edema
Hyperemic fillings out
Ordinary color, fillings out
Not changed
At what percent of fluid loss will be II degree of dehydration?
3-6 % of body weight
6-9 % of body weight
1-3 % of body weight
0,5-2 % of body weight
2-7 % of body weight
At what percent of fluid loss will be III degree of dehydration?
A.
B.
C. *
D.
E.
283.
A.
B.
C.
D. *
E.
284.
A.
B.
C.
D. *
E.
285.
A. *
B.
C.
D.
E.
286.
A. *
B.
C.
D.
E.
287.
A.
B.
C. *
D.
E.
288.
A.
B.
C. *
D.
E.
289.
A.
B.
C.
D.
3-6 % of body weight
Over 10 % of body weight
6-9 % of body weight
4-8 % of body weight
10-15 % of body weight
At what percent of fluid loss will be IV degree of dehydration?
4-8 % of body weight
6-9 % of body weight
3-6 % of body weight
Over 10 % of body weight
Over 15 % of body weight
B-12 deficiency is cause by which of the following:
Echinococus granulosis
T. saginata
E. multilocularis
Diphyllobothrium latum
Ascaris lumbricoideus
Balantidiasis is caused by:
B. coli
B. anthracis
E. coli
M. hominis
B. melitensis
Chyluria is the complication of
lymphatic filariasis
abdominal angiostrongyliasis
enterobiasis
trichuriasis
amebiasis
Cryptosporidiosis is:
blood borne infection
respiratory infection
intestinal infection
infection of external covers
helminthiasis
Cysticercosis belongs to:
Nematodosis
Trematodosis
Cestodosis
Ricketsiosis
Mycosis
Cycticercosis is:
Bacterial infection
Viral infection
Protozoosis
Fungal infection
E. *
290.
A.
B.
C. *
D.
E.
291.
A.
B.
C.
D.
E. *
292.
A.
B.
C. *
D.
E.
293.
A.
B.
C. *
D.
E.
294.
A.
B.
C.
D.
E. *
295.
A. *
B.
C.
D.
E.
296.
A.
B.
C.
D.
E. *
297.
A.
B.
Helminthosis
Diphyllobothriosis belongs to:
Nematodosis
Trematodosis
Cestodosis
Ricketsiosis
Mycosis
Diphyllobothriosis is:
Bacterial infection
Viral infection
Protozoosis
Fungal infection
Helminthosis
Drug of choice for the treatment of the lymphatic filariasis is:
albendazole
steroids
diethylcarbamazine
metronidazole
chloramphenicol
Echinococcosis belongs to:
Nematodosis
Trematodosis
Cestodosis
Ricketsiosis
Mycosis
Echinococcosis is:
Bacterial infection
Viral infection
Protozoosis
Fungal infection
Helminthosis
Enterobiosis belongs to:
Nematodosis
Trematodosis
Cestodosis
Ricketsiosis
Mycosis
Enterobiosis is:
Bacterial infection
Viral infection
Protozoosis
Fungal infection
Helminthosis
Teniosis belongs to:
Nematodosis
Trematodosis
C. *
D.
E.
298.
A.
B.
C.
D.
E. *
299.
A.
B.
C. *
D.
E.
300.
A.
B.
C.
D.
E. *
301.
A. *
B.
C.
D.
E.
302.
A.
B.
C.
D.
E. *
303.
A. *
B.
C.
D.
E.
304.
A.
B.
C.
D.
E. *
305.
Cestodosis
Ricketsiosis
Mycosis
Teniosis is:
Bacterial infection
Viral infection
Protozoosis
Fungal infection
Helminthosis
Teniarinchosis belongs to:
Nematodosis
Trematodosis
Cestodosis
Ricketsiosis
Mycosis
Teniarinchosis is:
Bacterial infection
Viral infection
Protozoosis
Fungal infection
Helminthosis
Strongiloidosis belongs to:
Nematodosis
Trematodosis
Cestodosis
Ricketsiosis
Mycosis
Strongiloidosis is:
Bacterial infection
Viral infection
Protozoosis
Fungal infection
Helminthosis
Ancilostomosis belongs to:
Nematodosis
Trematodosis
Cestodosis
Ricketsiosis
Mycosis
Ancilostomosis is:
Bacterial infection
Viral infection
Protozoosis
Fungal infection
Helminthosis
Etiology agent of botulism is:
A.
B.
C. *
D.
E.
306.
A.
B.
C. *
D.
E.
307.
A.
B. *
C.
D.
E.
308.
A.
B.
C.
D.
E. *
309.
A.
B.
C.
D. *
E.
310.
A.
B.
C.
D. *
E.
311.
A.
B.
C. *
D.
E.
312.
A. *
B.
C.
Campylobacter
Balantidium coli
Cl. botulinum
Cl. perfrigens
S. aureus
Etiology agent of botulism is:
Ch. trachomatis
Escherichia coli
Cl. botulinum
Cl. perfrigens
Rotavirus
Fasciolosis belongs to:
Nematodosis
Trematodosis
Cestodosis
Ricketsiosis
Mycosis
Fasciolosis is:
Bacterial infection
Viral infection
Protozoosis
Fungal infection
Helminthosis
For the rehydration in dehydration shock it is necessary to conduct the permanent careful account of
all losses of liquid in each:
4 hrs
30 hrs
3 hrs
2 hrs
5 hrs
For verification of diagnosis of balantidiasis more frequently used test is:
Virological researches
Bacteriological examinations
Roentgenologic researches
Research on protozoa
Ultrasound
How long does last health centre system of convalescent after balantidiasis?
6 months
3 months
1 year
2 years
5 years
How often treatment of amoebae cyst carrier should be done?
Twice a year
Three times a year
One time a year
D.
E.
313.
A.
B.
C.
D. *
E.
314.
A.
B.
C. *
D.
E.
315.
A.
B.
C.
D. *
E.
316.
A.
B.
C.
D.
E. *
317.
A.
B.
C. *
D.
E.
318.
A. *
B.
C.
D.
E.
319.
A.
B. *
C.
D.
E.
Does not treat
Quarterly
How to increase frequency of findings of lamblias cyst in fresh feces and vegetative forms in
duodenal content?
Cultivation in thermostat
By the method of floatation in bilious clear soup
Cultivation in anaerobic chamber
By the applications of phase-contrast and luminescent microscopy with the help of methylen-orange
To cultivate on a nourishing environment
In 1 liter of Trisalt solution, the concentration of potassium chloride is:
3 g/l
1.5 g/l
1.0 g/l
2 g/l
2.5 g/l
In a settlement was found out a few cases of cholera. Who must be insulated?
Persons with dysfunction of intestine
Patients with cholera
Carriers
Persons contact with the sick patient
Persons with hyperthermia
In the break out of cholera it is necessary to carry out such measures, except:
Hyperchlorination of drinking water
An active discovery of patients by rounds
Obligatory hospitalization, inspection and treatment of patients and vibrio tests
Revealing and isolation of contact persons
Vaccine prophylaxis
In the different places of settlement found out a few cases of disease of cholera. Who from the
contacts of cholera patient is sent in an insulator?
Vibrio positive
Patients with cholera
Contact with the patient persons
Persons with dysfunction of intestine
Persons with high temperature
Loffler syndrome at Ascariasis is due to
inflitration of lung tissue by eosinophills
inflitration of payer's patches by eosinophills
invasion of gallblader by A.lumbricoides pathogen
inflitration of liver by eosinophills
none of enumerated
Child, 5 years, itching in the perianal region. Most probable diagnose will be:
Trichinosis
Enterobiosis
Ascaridosis
Helminthosis
Cystitis
320.
A.
B. *
C.
D.
E.
321.
A.
B. *
C.
D.
E.
322.
A.
B.
C.
D.
E. *
323.
A.
B.
C.
D. *
E.
324.
A.
B. *
C.
D.
E.
325.
A.
B.
C. *
D.
E.
326.
A.
Onchoceriasis is also known as:
tropical pulmonary eosinophillia
river blindness
guinea worm infection
African eye worm disease
ricketsiosis
Opisthorchosis belongs to:
Nematodosis
Trematodosis
Cestodosis
Ricketsiosis
Mycosis
Opisthorchosis is:
Bacterial infection
Viral infection
Protozoosis
Fungal infection
Helminthosis
Patien O., 29 years old, farmer appeared to a doctor with the signs of balantidiasis and was
hospitalized. The best etiotropic drug is:
Benzylpenicillin
Gentamycin
Levomycitin
Monomycin
Timogen
Patient D., 13 y.o., hospitalized with complaints of nausea, abdominal pain, liquid emptying without
pathological admixtures with an unpleasant smell, 6 times per day. The day before he had a supper
with meat salad. What is a first aid?
Peroral rehydration by glucose-electrolytes solutions
To wash a stomach and intestine by solution of hydrocarbonate of sodium
Antibacterial preparations of wide spectrum of action
Renewal of normal microflora of intestine
To wash a stomach and intestine by solution of permanganate of potassium
Patient H., 37 y.o., appealed to the doctor on the second day of disease with complaints of the
promoted fatigueability, weakness in muscles („cotton feet”), violation of sight, dryness of mouth.
Works in a tourist agency, often is in the oversea business trip. Three days ago was with friends on a
picnic in a forest, where ate the varied meal of the domestic making. At a review: Т-36,8 °C.
Blepharoptosis. Midriasis. The reaction on light is lost. A soft palate is not changed. Defecation is
absent for 2 days. What is the source of infection in this disease?
The sick man
Man-bacteria carrier
Domestic animals
Canned foods
Ticks
Patient I., 34 years old, entered to the infectional department. She is sick during 4 days. She
complained on binocular diplopia, “rate” in front of eyes, erostomya, dysphagia, myastenia. What is
previous diagnosis?
Stool
B.
C. *
D.
E.
F.
G.
H.
I.
J.
327.
A.
B.
C. *
D.
E.
328.
A.
B.
C.
D.
E. *
329.
A.
B.
C.
D. *
E.
330.
A.
B.
C.
D. *
E.
331.
A.
B.
C.
D.
Wine
Blood
Vomiting mass
All above it
Poisoning by Belladonna
Diphtherial polyneuritis
Botulism
Rotaviral infection
Poliomyelitis
Patient I., 34 years old, entered to the infectious department. She is sick during 4 days. She
complained on binocular diplopia, “net” in front of eyes, xerostomya, dysphagia, myastenia. What is
previous diagnosis?
Poisoning by Belladonna
Diphtherial polyneuritis
Botulism
Rotaviral infection
Poliomyelitis
Patient M, 32 y.o., entered clinic on 3rd day of disease with complaints of nausea, feeling of weight
in abdomen, vomits, liquid stool. Then appeared clouds before eyes, doubling of objects, voice
hoarse, violation of swallowing. A day before he used the dried fish. During review: state is
moderate., violation of active motions like paralyses. There is also midriasis, vertical nystagmus,
blepharoptosis, absent reaction of pupils on light. Tongue is dry. Flatulence of 2 stage. What methods
of laboratory diagnostics are used to confirm the diagnosis?
Reaction indirect hemaglutination
Immunofluorescent
Virology
Reaction of coaglutination
Reaction of neutralization (biological test)
Patient M., 35 years old, who is sick during 2 days, complains on xerostomia, dysphagia. What
symptom is necessary to check?
The Padalra‘s symptom
The Stefansky‘s symptoms
The corneal symptoms
The eyes symptoms.
All above it
Patient M., 35 years old, who is sick during 2 days, complain on herostomia, dysphagia. What
symptom is necessary to check?
The Padalra‘s symptom
The Stefansky‘s symptoms
The corneal symptoms
The eyes symptoms.
All above it
Patient with cholera has bradycardia, low blood pressure, weakness. What is most important factor in
the given clinical situation?
Hypocalcemia
Hypopotassium
Hyponatremia
Hypernatremia
E. *
332.
A.
B. *
C.
D.
E.
333.
A.
B.
C.
D. *
E.
334.
A.
B.
C. *
D.
E.
335.
A.
B.
C.
D. *
E.
336.
A.
B. *
C.
D.
E.
337.
A.
B.
C.
D.
E. *
338.
A. *
B.
Hyperpotassium
Patient with cholera on a background treatment has signs of hyperkalemia. What solution must be
applied for futher rehydration therapy?
Neohemodez
Disalt
Chlosalt
Polyhybrid
Lactosalt
Patient, 32 y.o., complains of progressing muscular weakness, worsening of sight, doubling of
objects, “net”, before eyes, violation of swallowing (can not swallow a hard meal), thirst. In the first
day of illness single liquid stool was present without pathological admixtures, nausea. 2 days prior to
beginning of disease was in guests, used an alcohol, canned mushrooms. Presence of ptosis,
midriasis, anizocoria. Tones of heart are muffled. Which antibotulinic serum is it expedient to enter?
Mixture of serums of types A and E for 5 thousands IU and type B 10 thousands of IU
Mixture of serums of types A, B and E for 10 thousands of IU
Mixture of serums of types A, B and E for 5 thousands of IU
Mixture of serums of types A and E for 10 thousands IU and type B 5 thousands of IU
Mixture of serums of types A and B for 10 thousands IU and type E 5 of thousand of IU
Patient, 40 y.o. in 5 hours after the use in the meal of canned mushrooms of the domestic making a
sharp general weakness, nausea, vomits, dryness of mucus membranes of oral cavity, doublings of
objects, disorders of act of swallowing. Diagnosis?
Poisoning by Belladonna
Diphtherial polyneuritis
Botulism
Rotaviral infection
Poliomyelitis
Preparation of choice for the treatment of carrier of cyst of amoebae is:
Monomycinum
Delagilum
Tetracyclin
Yatrenum
Ursosan
Schistosomosis belongs to:
Nematodosis
Trematodosis
Cestodosis
Ricketsiosis
Mycosis
Schistosomosis is:
Bacterial infection
Viral infection
Protozoosis
Fungal infection
Helminthosis
Strongyloidosis belongs to:
Nematodosis
Trematodosis
C.
D.
E.
339.
A.
B.
C.
D.
E. *
340.
A.
B.
C. *
D.
E.
341.
A.
B.
C.
D.
E. *
342.
A.
B.
C. *
D.
E.
343.
A.
B.
C.
D.
E. *
344.
A.
B.
C.
D. *
E.
345.
A. *
B.
C.
D.
E.
346.
Cestodosis
Ricketsiosis
Mycosis
Strongyloidosis is:
Bacterial infection
Viral infection
Protozoosis
Fungal infection
Helminthosis
Teniarinchosis belongs to:
Nematodosis
Trematodosis
Cestodosis
Ricketsiosis
Mycosis
Teniarinchosis is:
Bacterial infection
Viral infection
Protozoosis
Fungal infection
Helminthosis
Teniosis belongs to:
Nematodosis
Trematodosis
Cestodosis
Ricketsiosis
Mycosis
Teniosis is:
Bacterial infection
Viral infection
Protozoosis
Fungal infection
Helminthosis
The balantidiasis possible complications are all, except:
Hypochromic anaemia
Enterorrhagia
Perforation of ulcer
Abscess of liver
Cachexia
The clinical forms of balatidiasis are all, except
Lightning rapid
Acute
Subclinical
Chronic continues
Chronic recurrent
The distinctive pattern of movement of filarial worms in lymphatic vessels is known as:
A.
B. *
C.
D.
E.
347.
A.
B.
C.
D. *
E.
348.
A.
B.
C.
D.
E. *
349.
A. *
B.
C.
D.
E.
350.
A.
B.
C.
D.
E. *
351.
A.
B.
C. *
D.
E.
352.
A. *
B.
C.
D.
E.
353.
filaria jumping sign
filaria dance sign
filaria swim sign
filaria escape sign
filaria flying sign
The most effective means of filariasis control will be:
mass yatren therapy
insecticidal measures against culex mosquitoes
provision of underground drainage
personal prophylaxix
all mentioned above
The patient, 45 y.o., entered clinic on the 2nd day of illness with complaints of a weakness, diplopia,
dryness in mouth, constipations. 3 days ago ate the smoked pork of the domestic making. At a
review: skin pale, consciousness is clear. Temperature 37,2 C, pulse 68 in 1 min, AP 120/80, pupils
are widening, reaction on light slow, ptosis, horizontal nystagmus. Paresis of soft palate. A
sensitiveness is normal. Meningeal signs are not present. The most effective treatment will be:
Sulfanilamides
Antibiotics
Salts solutions
Antiviral preparations
Antibolutilic antitoxic serum
Toxocarosis belongs to:
Nematodosis
Trematodosis
Cestodosis
Ricketsiosis
Mycosis
Toxocarosis is:
Bacterial infection
Viral infection
Protozoosis
Fungal infection
Helminthosis
Trichinellosis develops after:
bite of a tick
drinking of contaminated water
ingestion of the infected meat
bite of a dog
all of the above
Trichinosis belongs to:
Nematodosis
Trematodosis
Cestodosis
Ricketsiosis
Mycosis
Trichinosis is:
A.
B.
C.
D.
E. *
354.
A. *
B.
C.
D.
E.
355.
A.
B. *
C.
D.
E.
356.
A. *
B.
C.
D.
E.
357.
A.
B.
C. *
D.
E.
358.
A.
B.
C. *
D.
E.
359.
A.
B.
C.
D.
E. *
360.
A. *
B.
C.
Bacterial infection
Viral infection
Protozoosis
Fungal infection
Helminthosis
What clinically active forms of cholera do you know?
Very rapid acute for the children and elderly persons
“Choleric typhoid”, acute subclinical, for the children and elderly persons
Dry, very rapid, “choleric typhoid”, subclinical for the children and elderly persons
Very rapid “choleric typhoid”, acute, subclinical, for the children and elderly persons
Very rapid, dry, subclinical, for the children and elderly persons
What from the below mentioned preparations can be used for the treatment of primary rehydration?
Lactosalt
Disalt
Acesalt
Trisalt
Khlosalt
What from the below mentioned preparations, can be used for the treatment of primary rehydration?
Polyhybrid
Acesalt
Khlosalt
Kvartasalt
Lactosalt
What from the below mentioned preparations, can be used for the treatment of primary rehydration?
Acesalt
Trisalt
Cryoplasma
Khlosalt
Lactosalt
What is larva currents
dead larva
floating larva
running larva
slipping larva
none of the above
What is the main symptom of the Trichinellosis:
Rash
Muscle pain
Edema of eyelids
Nodules in muscles
All mentioned above
What kind of ulcers are present at аmebiasis?
Fillings out sharp edges, surrounded by the area of hyperemia, are placed on the unchanged mucus
membrane
Smooth sharp edges, placed on a hyperemic mucus membrane
Even edges, placed on a hyperemic mucus membrane
D.
E.
361.
A.
B.
C.
D.
E. *
362.
A. *
B.
C.
D.
E.
363.
A. *
B.
C.
D.
E.
364.
A.
B.
C.
D. *
E.
365.
A. *
B.
C.
D.
E.
366.
A. *
B.
C.
D.
E.
367.
A.
B.
Fillings out sharp edges, placed on the unchanged mucus membrane
Fillings out sharp edges, surrounded by the area of hyperemia, are placed on the unchanged mucus
membrane
What material should be taken to discharge the Cl. Botulinum?
Stool
Food debris
Blood
Vomiting mass
All above it
What special treatment is used in beginning of the botulism?
Antibotulinum serum
Disintoxication therapy
Hormonal therapy
Sulfonamides therapy
Vaccine therapy
What special treatment used in beginning of the botulism?
Antibiotic therapy
Disintoxication therapy
Hormonal therapy
Sulfonamides therapy
Vaccine therapy
What time is it necessary to complete primary rehydration at dehydration shock?
3-5 hrs
0.5 hrs
2-3 hrs
1-1.5 hrs
4-6 hrs
When it is.possible to write reconvalensense state of аmoebiosis from permanent establishment?
After clinical convalescence, in default of in incandescence of mucous, еosinophils, crystals of
Charkot-Leiden and two negative results of parasitological research of excrements
After clinical convalescence, in default of in incandescence of blood and three negative results of
parasitological research of excrements
After clinical convalescence, in default of leukocytosis and two negative results of parasitological
research of excrements
After clinical convalescence and normalization of indexes of global analysis of blood
After clinical convalescence, normalization of indexes of global analysis of blood and two negative
results of bacteriological examination of excrement
When would you discharge a patient from hospital, who was diagnosed with balandiasis?
After clinical recovery and two negative results of parasitological research of excrement
After clinical recovery and two negative results of bacteriological examination of excrement
After clinical recovery and one negative result of parasitological research of excrement
After clinical recovery and normalization of indexes of global analysis of blood
After clinical recovery, normalization of indexes of global analysis of blood and two negative results
of bacteriological examination of excrement
Which drug can be used in pregnancy in case of ascariasis?
albendazole
mebendazole
C.
D.
E. *
368.
A.
B.
C.
D. *
E.
369.
A.
B. *
C.
D.
E.
370.
A.
B. *
C.
D.
E.
371.
A. *
B.
C.
D.
E.
372.
A. *
B.
C.
D.
E.
373.
A.
B.
C.
D. *
E.
374.
A.
B.
C. *
D.
E.
375.
pyrantel pamoate
ivermectin
piperasin adipinat
Which from the below is a complication of cholera?
Collapse
Infectious-toxic shock
Acute renal insufficiency
Dehydration shock
Status typhosis
Which groups of symptoms are occurs in the clinic of botulism?
Vomiting, higher temperature
Dysphagia, dysphonia, diplopia,
Sickness, general weakness
Higher temperature, diarrhea, speech dysfunction
Diarrhea, vomiting dysfunction of eyesight
Which groups of symptoms are occurs in the clinic of botulism?
Vomiting, higher temperature
Dysfunction of speech and eyesight, breath, sickness, dysphagia
Sickness, general weakness
Higher temperature, diarrhea, speech dysfunction
Diarrhea, vomiting dysfunction of eyesight
Which of the following is known as pinworm
E. vermicularis
A. duodenale
N. americanus
T. solium
all of the above
Which of the following is the largest intestinal helminthes in human:
D. latum
S. stercoralis
Anisakis simplex
E. vermicularis
T. saginatus
Which of the following species of Trichinella are distributed world wide:
T.nelsoni
T.spiralis
T.nativa
All mentioned above
None
Which salt solutions do not contain potassium?
Trisalt
Lactosalt
Disalt
Qudrosalt
Khlosalt
Who must be admitted in the hospital from the focus of cholera?
A.
B.
C. *
D.
E.
376.
A. *
B.
C.
D.
E.
377.
A. *
B.
C.
D.
E.
378.
A. *
B.
C.
D.
E.
379.
A.
B. *
C.
D.
E.
380.
A.
B.
C. *
D.
E.
Carriers
Patients with cholera
Persons with disfunction of intestine
Contact persons
Persons with high temperature
With the purpose of specific prophylaxis of cholera is used:
Cholerogen-toxoid
Vaccine
Nitrofuranes
Immunoprotein
Antibiotics
With which serum reactions it is possible to confirm the diagnosis of balantidiasis?
Complement link reaction, reaction in gel precipitation, reaction of immobilization
Reaction of indirect gemagglutination, immune fluorescent reaction
Complement link reaction, reaction of indirect gemagglutination
Complement link reaction, immune fluorescent reaction, reaction of indirect gemagglutination
Complement link reaction, reaction of indirect gemagglutination
Woman L, 65 y.o., became ill sharply, in 12 hours after the use in the meal of canned mushrooms of
the domestic making and fried eggs fried on fat. A sharp weakness, nausea, double vomits, appeared
„clouds” before eyes, disorders of swallowing. At a review: Т-36,2 C., ptosis, midriasis,anizocoria,
inspiratory dyspnea. What disease is present in women?
Botulism
Salmonelosis
Poisoning by mushrooms
Sharpening of chronic cholecystitis
Toxic food-borne infection
Two girls came to a hospital, because they had 38 0С fever, a headache, weakness, dizziness, and a
pain in epigastry and round a navel, a nausea, vomiting 3 times, excrements 4 times per a night,
watery diarrhea, foamy, fetid, with mucus impurity. It is known from the history that the day before
the girls ate pastries with cream which were not stored in a refrigerator. Objectively: a tongue is dry,
furred by white touch, the stomach is bloated moderately, rumbles in palpation, painful in epigastry,
pulse is 80 bpm, and the blood pressure is 110/70 mm mercury column. To define the diagnosis.
Shigellosis
Salmonellosis
Food poisoning
Typhoid fever
Cholera
Two girls came to a hospital, because they had 38 0С fever, a headache, weakness, dizziness, and a
pain in epigastry and round a navel, a nausea, vomiting 3 times, excrements 4 times per a night,
watery diarrhea, foamy, fetid, with mucus impurity. It is known from the history that the day before
the girls ate pastries with cream which were not stored in a refrigerator. Objectively: a tongue is dry,
furred by white touch, the stomach is bloated moderately, rumbles in palpation, painful in epigastry,
pulse is 80 bpm, and the blood pressure is 110/70 mm mercury column. To make the treatment plan.
Diet 5, regidron, polifepan, enzymes, nifuroksazyd
Diet 5, regidron, polifepan, enzymes, furagin
Diet 5, regidron, polifepan, enzymes
Diet 5, regidron
Diet 5, polifepan, enzymes, furagin
381.
A. *
B.
C.
D.
E.
382.
A.
B.
C.
D. *
E.
383.
A.
B.
C. *
D.
E.
384.
A. *
B.
C.
D.
E.
385.
A.
B.
C. *
D.
E.
386.
A. *
B.
C.
D.
A 23 years old person, became ill sharply: fever 38.2 °C, moderate diffuse pharyngalgia at
swallowing, pain and itching in the right eye. Objectively: tonsillitis, pharyngitis, conjunctivitis.
What is previous diagnosis?
Adenoviral infection
Enteroviral infection
Parainfluenza
Flu
Acute respiratory infection
A 25 years old patient, fell ill rapidly, with chills and temperature rose to 39,9 оC, headache appeared
in frontotemporal regions, pain in eyeballs, dull pain in all trunk, closed nose. Dry cough after 2 days
of illness, there was nose bleeding. Objectively: hyperemia with sputum, isolated petechial rash and
shallow grittiness of soft palate. Difficult breathing in lungs. What is the most possible diagnosis?
Leptospirosis
Adenoviral infection
Typhoid fever
Flu
Epidemic typhus
A 4 years old child complains about: cough, temperature of body 38.1 °C. Conjunctiva is hyperemic.
On mucous of cheeks there are points of hyperemia gum blushs. Weaken breathing in the lungs.
What is the most possible diagnosis?
Scarlet fever
Rubella
Measles
Herpetic infection
Flu
A child 10 years old with temperature 38,0 °C, conjunctivitis, moist cough, hyperemia of the mucous
membranes of cheeks and lips. Gums are pallor. What is your diagnosis?
Measles
Adenoviral infection
Acute respiratory viral infection
Enteroviral infection
Infectious mononucleosis
A child 3 years old is found in the grave condition – naughty, forced breathing, dry «barking»
coughing, voice is hoarse, perioral cyanosis. The third day, temperature of body is subfebrile, mild
common cold. In lungs single dry wheezes can be heard. Moderate tachycardia. For which disease
these symptoms are characteristic?
Localized diphtheria of oropharynx
Whooping-cough
Parainfluenza, false croup
Bronchopneumonia
Adenoviral infection
A child of age 2 years has temperature of body 37.3 °C, cold, hoarse voice “barking cough” appeared
suddenly the anxiety, shortness of breath, appeared with participation of auxiliary muscles. Supposed
diagnosis?
Parainfluenza, false croup
Diphtheria croup
Allergic laryngitis, croup
Flu, laryngitis
E.
387.
A. *
B.
C.
D.
E.
388.
A.
B.
C. *
D.
E.
389.
A. *
B.
C.
D.
E.
390.
A.
B.
C. *
D.
E.
391.
A.
B. *
C.
D.
E.
392.
A. *
Acute exudative pleuritis
A normal indices of impalpable fluid losses of the adult person with 70 kg body weight is:
1000 ml per a day
500 ml per a day
700 ml per a day
1500 ml per a day
2000 ml per a day
?A pathological state which develops owing to catastrophic reduction of a circulating fluid volume
and electrolytes loss is:
An infectious-toxic shock
An anaphylactic shock
A dehydrationous shock
A hemorrhagic shock
All right
A patient 14 years old, hospitalized in the infectious department in severe condition with
considerable headache mainly in frontal and temporal area, pain in eyeballs, in muscles and joints.
Objectively: patient is excited, temperature of the body is 39 оC. Bradycardia changed by
tachycardia. Muscles tonic and clonic cramps. Positive meningeal signs. It is found in epidemic
anamnesis, his brother is also sick. What is your diagnosis?
Flu with pneumonia and edema of brain
Flu, typical course
Parainfluenza, false croupe
Respiratory-sencytial infection
Adenoviral infection, pneumonia
A patient 17 years old, became suddenly ill: temperature rose to 40,3 °C. Severe headache, motive
excitation, frequent vomiting, tremor of fingers of extremities. Hemorrhagic spots of round form and
different sizes, more frequently as stars, mainly on buttocks and trunk. Meningeal signs are positive.
What is the most possible diagnosis?
Encephalitis
Flu with a hemorrhagic syndrome
Meningococcal infection
Measles
Leptospirosis
A patient 17 years, 11th class student, were a lot of cases of ARI (acute respiratory infection) have
happened, appealed to a doctor in clinic at 3rd day of disease with complaints of chills, general
weakness, a moderate sore throat, running nose, swelling of face, watering from eyes Objective
examination: minor palatal hyperemia brackets and tonsillitis, on a background of moderate edema of
tissues. Conjunctivitis. During palpation not painful enlarged inframaxillary lymph nodes, and
enlarged neck lymph nodes were found. Crepitation can’t be find. Liver and spleen moderately
increased. What is the most likely diagnosis?
Diphtheria
Adenoviral infection
Meningococcal nasopharyngitis
Influenza
Infectious mononucleosis
A patient 18 years old, with complaints about headache, pharyngalgia, weakness, high temperature.
Objectively: all groups of lymphonodes, 1-3 cm in a diameter, dense, elastic, enlarged,
hepatospleenomegaly. Blood analysis: leukocytosis, mononuclear – 15 %. What is possible
Infectious mononucleosis
diagnosis?
B.
C.
D.
E.
393.
A. *
B.
C.
D.
E.
394.
A. *
B.
C.
D.
E.
395.
A.
B. *
C.
D.
E.
396.
A. *
B.
C.
D.
E.
397.
A. *
B.
C.
D.
Adenoviral infection
Angina
Diphtheria
Acute lympoleycosis
A patient 20 years old fell ill rapidly with increasing of temperature to 39.9 °C. complains about
headache in frontotemporal region, pain in eyeballs, dull ache in whole trunk, closed nose, scrapes in
the throat, dry cough. There was nose-bleeding. What diagnosis is most possible?
Influenza
Adenoviral infection
Parainfluenza
RS-infection
Enteroviral infection
A patient 20 years old, complains about increasing of temperature up to 39 оC, headache in frontal
area, pain in eyeballs, photophobia, pain in muscles, dry cough. Became ill sharply. Objectively:
severe state. Face is hyperemic, eyes brilliant, injections of scleras. Pulse 96/min, rhythmic, tones of
heart are hyposthenic. Menengial symptoms are not present. Blood analysis: leuk 9?109, е 1 %,
bands 6 %, seg 51 %, lymp 35 %, mono 7 %. What is the most possible diagnosis?
Ifluenza
Adenovirus infection
Leptospirosis
Pneumonia
Epidemic typhus
A patient 26 years old, became sick rapidly: temperature 39.5 оC, severe headache, mainly in frontal
and temporal areas, pain in muscles and joints. Examined on the 2nd day of illness: state of middle
weight, skin is clean. Moderate hyperemia with cyanosis, pulse 120 per min, rhythmic. Heart activity
is rhythmic, tones are muffled, in lungs there is vesicular breathing. What is the treatment of this
Aspirin
patient?
Remantadin
Ampicillin
Ascorbic acid
Ribonuclease
A patient 27 years old, entered clinic on the 4th day of illness with a diagnosis ARVI, allergodermia.
Fell ill with the rise of temperature to 38,0 °C, headache, hyperemia of the throat, then barking cough
appeared. On the 3rd day rash appeared on the skin and neck. Was treated by aspirin. Objectively:
temperature 38.8 °C. Face is puffy, conjunctivitis. On the skin of neck and upper part of chest is
abundant red-papular rashes as rings which does not itch. Mucosa of epiglottis is brightly hyperemic.
Submandibular and neck lymphadenitis. Liver and spleen were not enlarged. What is your diagnosis?
Measles
Allergic dermatitis
Infectious mononucleosis
German measles
Scarlet fever
A patient 52 years old, hospitalized with the severe form of viral hepatitis B. The signs of flu
appeared in the department. The indexes of bilirubin rose up and transaminase falls down. What
complication can arise in that patient?
Acute hepatic insufficiency
Infectious-toxic shock
Gastric bleeding
Neurotoxicosis
E.
398.
A.
B.
C.
D. *
E.
399.
A.
B.
C.
D.
E. *
400.
A.
B. *
C.
D.
E.
401.
A.
B.
C. *
D.
E.
402.
A.
B.
C.
D. *
E.
Cerebral comma
A patient 56 years old, the day before felt easy indisposition, insignificant headache, and weakness.
Afterwards the increasing of temperature appeared to 38,5 оC with chills, headache increased
considerably, mainly in forehead and temples. Skin and conjunctiva is hyperemic, dry, barking
cough. Pharynx is hyperemic. On soft palate present grainy granules, placed point hemorrhages.
Difficult breathing. What is the most possible diagnosis?
Typhoid fever
Leptospirosis
Epidemic typhus
Flu
Enteroviral infection
A patient 56 years old, workwomen of pig farm, on a background chills appeared, the temperature
rose to 39,9 °C, headache, nausea. The next day marked pains in the muscles of lower extremities
appeared, the nose bleeding began. At the receipt, on the 3rd day common state deteriorated.
Hyperemic spots, subecteric appeared. Liver +3 cm. Daily – diuresis 700 ml. What is the previous
Hemorrhagic
diagnosis? fever with a kidney syndrome
Hepatitis A
Escerichiosis
Flu
Leptospirosis
A patient A., 30 years old, on the 4th day of illness a district doctor marked such subjective and
objective data: insignificant indisposition, mild headache, hoarseness of voice, itching in throat,
breakingdry cough, temperature of the body 37,4 °C. Pulse 86/min., difficult nasal breathing,
insignificant serous excretions from nose. Which acute respiratory infection does the patient carry?
Influenza
РC-viral infection
Parainfluenza
Adenoviral infection
Enteroviral infection
A patient admitted in the infectious department with diagnosis of acute respiratory viral infection.
Became ill suddenly, the disease is accompanied by the increase of temperature of body till 39 °C, by
severe headache, mainly in area of frontal, temporal, above eyes, dryness in nose, itching in throat,
dry cough, dull pain in all body. He had bleeding from nose twice at home. Which acute respiratory
disease has the patient?
Adenoviral infection
РC-infection
Flu
Parainfluenza
Enteroviral infection
A patient becomes sick very fast: chills, increasing of temperature to 40,1 оC, headache in frontal and
temporal regions, pain in eyeballs, close nose, dry cough and pain in the chest. The nose bleeding,
nausea, double vomits. Objectively: conjunctivitis, hyperemia, edema, hemorrhages in mucous of
otopharhynx, tachycardia. Blood pressure is low. Difficult breathing . What is the most possible
diagnosis?
Meningococcemia
Epidemic typhus
Leptospirosis
Flu
Typhoid fever
403.
A.
B.
C.
D.
E. *
404.
A.
B.
C.
D. *
E.
405.
A. *
B.
C.
D.
E.
406.
A.
B.
C. *
D.
E.
407.
A.
B. *
C.
A patient C., 25 years old, fell suddenly ill. Every morning severe headache, frequent vomiting,
temperature of the body is 39.9 °C. Adopted fatigue, then state got much worse. In the evening lost of
consciousness. Expressed muscles pains of back and head. Positive Кеrning’s symptom. Leukocytes
– 18,0?109. What is the most reliable diagnosis?
Flu
Epidemic typhus, typhus state
Viral menigoencephalitis
Sepsis, infectious-toxic shock
Bacterial menigoencephalitis
A patient C., was hospitalized on the 2nd day of illness with complaints about hoarseness of voice,
rough barking cough, labored breathing. Objectively: the state is severe, uneasy, pallor, temperature
37.1 °C, BR 30/min., breathing is noisy, can hear from the distance, with participation of auxiliary
musculature. Which viruses could cause development of similar state?
Rhino virus
Influenza virus
Adenovirus
Parainfluenza virus
Cytomegalovirus
A patient caused a doctor home. Age – 75 years. Complaints of a subfebrile temperature, general
weakness, pharyngalgia, conjunctivitis. In family a child is ill the acute adenoviral disease. A patient
considers itself a patient the second day. At a review are found out the signs of acute
blepharoconjunctivitis, pharyngitis. There are megascopic lymphatic knots: neck front and back,
arm-pits and inguinal, to 1 sm in a diameter, soft, not is soldered between itself and with a
surrounding cellulose. A pharynx is hyperemic, tonsills are hypertrophied and hyperemic. In lights of
wheezes it is not. Breathing clean. Tones of heart are muffled. BP is 140/80 mm Hg. Ps – 80 per 1
minute. Stomach soft. A megascopic liver which comes forward on 3 cm below costal arc and spleen
are palpated – soft, painless. Choose the most credible diagnosis:
Acute adenoviral infection
Flu
Megacaryoblastoma
Infectious mononucleosis
Hepatitis A
A patient fell ill very rapidly: chills, increase of temperature to 40.1 °C, headache in frontotemporal
regions, pain in eyeballs, close nose, dry cough, and chest pain. Nose bleeding, nausea, vomiting
appeared after 4 hours. Objectively: conjunctivitis, hyperemia, edema, point hemorrhages in mucus
of epiglottis, tachycardia. Blood pressure is low. Weaken breathing in the lungs. What is the most
possible diagnosis?
Leptospirosis
Epidemic typhus
Flu
Мeningococcemia
Enteroviral infection
A patient H., 22 years old, with flu was hospitalized into infectious department with the acute
worsening of the common state. Consiousness is stored. The patient strangles. Pallor of skin with
cynosis. Respiratory rate 50 per min, AP 80/55 mmHg, pulse 110 per a min, temperature 39.8 оC.
During percussion of lungs tympanic sound with dullness in lower quadrant was found. Crackles in
the lower-back parts of lungs. What complication of influenza has developed in that patient?
Pneumonia
Edema of lungs
Edema of brain
D.
E.
408.
A. *
B.
C.
D.
E.
409.
A.
B.
C. *
D.
E.
410.
A. *
B.
C.
D.
E.
411.
A.
B. *
C.
D.
E.
412.
A. *
B.
C.
D.
E.
413.
A.
Infectious-toxic shock
Meningoencephalitis
A patient K., 23 years old, with 3 days of moderate illness, with high temperature of body to 40.0 °C,
headache and petechial rash on skin, is hospitalized. After introduction of penicillin at 2 o’clock, the
BP fell down to 40/10 mm of hg. Peripheral pulse and мeningeal signs does not concerne. What is the
diagnosis of the patient?
Меningococcemia, infectious-toxic shock
ARVI, anaphylactic shock
Measles, severe course
Epidemic typhus, severe course
Scarlet fever, severe course
A patient of 5 years old, which treated at home on an occasion of flu by aspirin, calcium gluconatis,
on the second day from the beginning of disease “coffee grounds” vomiting appeared, melena. What
complication arises?
Neurotoxicosis
Pneumonia
Hemorrhagic syndrome
Infectious-toxic
Bowel obstruction
A patient on the background of ARVI the fever developed to 40,1 °C, frequency of breathing is 40
for a minute. What measures are necessary?
Decreasing of patients temperature
Artificial ventillation
Oxygen. inhalation
Infusion therapy
Antibioticotherapy
A patient P., 14 years old, is hospitalized in the infectious dept. in grave condition. Complaints on
headache, mainly in frontal and temporal regions, superciliary arcs, vomiting on severe pain, pain on
movement of eyeballs, in muscles, joints. Objectively – a patient is excited, temperature of the body
39 °C. BP 100/60 mmHg. Bradycardia was replaced by tachycardia. Tonic cramps appeared.
Doubtful meningial signs. From anamnesis it is found that at home his brother has flu. What
preparations must be injected?
Verospiron, euphyllin, dimedrol
Mannitol, paracetamol, prednisolone, euphyllin
Analgin, dimedrol, aspirin, ampicillin
Mannitol, aspirin
Lasix, analgin, ampicillin
A patient with flu complicated by pneumonia, during some days there are the displays of
infectious-toxic shock of ІІ degree. In BA the level of urea and creatinine increases. What from these
preparations is not recommended to enter in such a situation?
Adrenalin
Prednisolone
Polioniic solutions
Dofaminum
Heparin
A patient with temperature of body 40.0 °C, nonproductive cough, photophobia, puffiness of face,
dots on gums, blushes on the mucus of cheeks your diagnosis?
Tuberculosis
B.
C. *
D.
E.
414.
A.
B.
C. *
D.
E.
415.
A. *
B.
C.
D.
E.
416.
A. *
B.
C.
D.
E.
417.
A.
B. *
C.
D.
E.
418.
A. *
B.
Меningococcemia
Measles
Enteroviral infection
Staphylococcal sepsis
A patient Т., 45 years old, was hospitalized at the 2nd day of disease. One week ago got back from
India (sailor of the distant swimming). Complains of temperature 41.3 °C, great headache, shortness
of breathing; cough with foamy pink colour sputum. Objectively: pale of face, cyanosis of mucous,
breath rate 24/min, tachycardia. Lungs: breathing is hyposthenic, moist wheezes in both lungs,
crepitation. What is possible diagnosis?
Flu
Miliary tuberculosis
Plaque, pulmonary form
Leptospirosis
Sepsis
A patient, 20 years old, during few days complains about pharyngalgias. After supercooling the state
became worse: sudden chills, increase of temperature to 40.6 °C, headache. On the skin of low
extremities, trunk and buttocks there are a lot of different sizes hemorrhagic spots, acrocyanosis.
Consiouness is preserved. Meningeal signs are absent. What is the previous diagnosis?
Meningococcal infection
Flu
Epidemic typhus
Hemorrhagic fever
Leptospirosis
A patient, 75 years old, called a doctor to home. Rashes and subfebrile temperature, general
weakness, pharyngalgia, conjunctivitis. In family a child is ill with acute adenoviral disease. A
patient considers himself ill on the second day. At a review there are signs of pharyngitis. There are
enlarged lymphatic nodes: of neck, front and back, armpits and inguinal up to 1 cm in diameter, soft.
Pharynx is hyperemic, tonsils are hypertrophy and hyperemic. Both lungs have wheezing sounds. Not
clean breathing. Tones of heart are muffled. AP 140/80 mm Hg. Heart rate 80 for 1 minute. Stomach
is soft. Enlarged liver 3 cm below costal arch and spleen is palpable. Palpation is soft, painless.
Acute adenoviral
infection
Choose
the most possible
diagnosis:
Flu
Hepatitis B
Infectious mononucleosis
Hepatitis A
A sick 15 years old, 3rd day of illness. On the background the catarrhal pneumonia, weakness in
hands appeared, double vision, cross-eye. Voice is weak. Palatoplegia and extended extremities.
Pulse 90/min. AP 130/90 mm Hg .What is your previous diagnosis?
Diphtheria
Poliomyelitis
Botulism
Epidemic encephalitis
Enteroviral infection
A sick 19 years old, fell ill rapidly, when a temperature rose to 39,2 °C, coughing appeared, closed
nose. Pains in muscles and joints. On the 3rd day of disease, shallow spots on the trunk appeared,
extremities with hyperemia and edematous feet. Generalized lymphadenopathy, hyperemia of cheeks,
enlargement of the liver were found out. What is previous diagnosis?
Pseudotuberculosis
Flu
C.
D.
E.
419.
A. *
B.
C.
D.
E.
420.
A. *
B.
C.
D.
E.
421.
A. *
B.
C.
D.
E.
422.
A. *
B.
C.
D.
E.
423.
A. *
B.
C.
Infectious mononucleosis
Herpetic infection
Epidemic typhus
A sick 70 years old, became ill sharply, the temperature of body rose to 39.2 °C, excited, euphoric,
hyperemia of face, Rozenberg’s exanthema appears. Ricketsia titer is 1:160, IgG – 87 %. What is
diagnosis?
Epidemic typhus
Meningococcal infection
Epidemic spotted fever
Flu
Parainfluenza
A sick explorer of train, 39 years old is hospitalized on the 4th day of illness with complaints about
headache, weakness, dizziness, chills, insomnia, fever. The person is hyperemic, conjunctivitis. On
the transitional fold of conjunctiva there is a single rash. On the skin of trunk, thorax, stomach,
extremities there are abundant red coloured rashes. Tachycardia. AP 100/60 mm of Hg. Tremor of
tongue. Liver and spleen were enlarged. Stool fistula is detained. What is the most reliable diagnosis?
Epidemic typhus
Typhoid
Flu
Меnigococcemia
Leptospirosis
A sick person, 45 years old, was hospitalized after 2 days of disease. On Sunday he came back from
India (sailor). Complains about increasing of temperature to 41 оC, severe headache, shortness of
breath, cough, with sputum. Objectively: pallor, cyanosis of mucous, tachycardia. Breathing is
weaken, crackles in the lower-back parts of the lungs, crepitation. What is the possible diagnosis?
Flu complicated by pneumonia
Miliary tuberculosis
Plague, pulmonary form
Leptospirosis
Sepsis
A sick woman, 42 years old, complaints about temperature 39.3 °C, headache in the frontal area, pain
in the eyeballs, photophobia, pain in muscles, dry cough. Became ill suddenly one day before.
Objectively: state is severe. Hyperemia of the face, eyes shinny, injection of scleras. Pulse 96/min.,
rhythmic. Tones of heart are hypotonic. Both lungs are dissipated. Dry wheezes. Mucosa of epiglottis
is hyperemic, grainy, vessels are extended. Meningeal symptoms are not present. Analysis of blood:
leuk – 3?109/l, еos – 1 %, band – 6 %, seg – 51 %, lymp – 35 %, mono – 7 %. What is the most
Flu
possible
diagnosis?
Measles
Meningococcal infection
Pneumonia
Epidemic typhus
A sick, 54 years old, hospitalized in infectious department in the grave condition. Complaints about
expressed headache, mainly in frontal and temporal areas superciliary arcs, origin of vomiting on
peak of pain. Objectively: patient is excited, temperature of body 39 оC, AP 100/60 mm Hg.
Bradycardia changed to tachycardia. Tonic cramps, meningeal signs appeared. From anamnesis it is
known that father is also sick. What treatment should be prescribed?
Mannitol, lasix, prednisolone, еuphyllin, suprastain
Mannitol, acetophene
Lasix, analgin, ampicillin
D.
E.
424.
A. *
B.
C.
D.
E.
425.
A.
B.
C.
D. *
E.
426.
A. *
B.
C.
D.
E.
427.
A. *
B.
C.
D.
E.
428.
A.
B. *
C.
D.
E.
429.
A.
B.
C. *
D.
E.
Verospiron, euphyllin, demidrol
Aspirin, analgin, demidrol
A student, 18 years old, for 7 days complains about weakness, hyperthermia to 37.8 °C, mucous
excretions from a nose, pharyngalgia at swallowing, pain in eyeballs. Objectively: increased lymph
nodes of neck and mandible, lymphadenitis, edema and injection of conjunctiva, hyperemia of
mucous of epiglottis, hypertrophy of tonsils. What is the most reliable diagnosis:
Adenoviral infection
Influenza
Infectious mononuleosis
Rhinoviral infection
Parainfluenza
A woman 27 years old, complaints about the general weakness, absence of appetite, coughing, fever
up to 37.5 °C for three weeks. Ulcerous illness of stomach, myocarditis is in anamnesis. What
inspection is primarily need to do?
Electrocardiography
Fibrobronchoscopy
Fibrogastroscopy
Fluorography
Common blood analysis
A woman who came back from a tour trip, the next day called emergency help. It is known from the
anamnesis, that within a week the temperature of body was moderately high. Complains of bad sleep
and bad appetite, pain in the stomach. During the assessment of the sick it is found out roseolas on
the pale skin of breasts and abdomen. Pulse is normal, temperature of body 38,2 °C,
hepatospleenomegaly. What is your previous diagnosis?
Typhoid fever
Epidemic typhus
Flu
Enteroviral infection
Leptospirosis
Amount of solutions necessary for the primary rehydration in cholera is.
Accordingly to the degree of dehydration at time of hospitalization
In accordance with the loss of liquid
2l
5 l|
10 l
Amount of solutions necessary for the secondary rehydration in cholera is.
Accordingly to the degree of dehydration at the time of hospitalization
In accordance with the loss of liquid
2l
5l
10 l
At a child with the clinical displays of ARVI a generilized lymphadenopathy, one-sided
conjunctivitis increase of liver and spleen, is marked. Most reliable diagnosis?
Infectious mononucleosis
Leptospirosis
Adenoviral infection
Flu
Pseudotuberculosis
430.
A.
B.
C. *
D.
E.
431.
A.
B.
C.
D.
E. *
432.
A.
B.
C.
D. *
E.
433.
A.
B. *
C.
D.
E.
434.
A. *
B.
C.
D.
E.
435.
A.
B.
C. *
D.
E.
436.
A.
B.
C.
D. *
E.
437.
At a child with the clinical displays of ARVI a generilized lymphadenopathy, one-sided
conjunctivitis increase of liver and spleen, is marked. Most reliable diagnosis?
Infectious mononucleosis
Leptospirosis
Adenoviral infection
Flu
Pseudotuberculosis
At I degree of dehydration the loss of liquid is:
0,5-1,5 % of body weight
6-9 % of body weight
3-6 % of body weight
5-8 % of body weight
1-3 % of body weight
At patient with pediculosis rapidly rise temperature of body up to 41.2 °C, headache, euphoria
appeared in 4 days from the beginning of illness. Red colour rash on the lateral thorax and back. Titer
of Rickettsia antibodies 1:640, Ig M – 89 %. What is diagnosis?
Flu
Enteroviral infection
Brill-Zinsser disease
Epidemic typhus
Parainfluenza
At what degree of dehydration, there will be “metabolic violation”:
Subcompensated
Negative
Irreversible
Moderate metabolic acidosis
Insignificant metabolic alkalosis
At what percent of fluid loss will be I degree of dehydration?
-6 % of body weight
6-9 % of body weight
1-3 % of body weight
0,5-2 % of body weight
2-7 % of body weight
At what percent of fluid loss will be II degree of dehydration?
3-6 % of body weight
Over 10 % of body weight
6-9 % of body weight
4-8 % of body weight
10-15 % of body weight
At what percentage of fluid loss will be IV degree of dehydration?
4-8 % of body weight
6-9 % of body weight
3-6 % of body weight
Over 10 % of body weight
Over 15 % of body weight
Can the symptoms of an acute appendicitis be the complications of typhoid fever:
A. *
B.
C.
D.
E.
438.
A.
B.
C.
D.
E. *
439.
A.
B.
C. *
D.
E.
440.
A.
B.
C. *
D.
E.
441.
A.
B. *
C.
D.
E.
442.
A. *
B.
C.
D.
E.
443.
A.
B. *
C.
D.
E.
444.
A.
B. *
C.
D.
Yes
No
Not always
Often
May be
Compensated dehydrationous shock develops:
Rise level of toxins
At a decrease of the systolic blood pressure
At a hyperthermia
At hypohemoglobinemia
Because of the haemodynamics changes absence in peace
Duration of isolation of patient with influenza complications?
4 days
7 days
10 days
17 days
20 days
Duration of isolation of patient with influenza complications?
4 days
7 days
10 days
17 days
20 days
Duration of therapy of primary rehydration in cholera is.
30 minutes
2 hours
6 hours
12 hours
1 days
Enterorrhagia feces:
Melena
Fetid
Does not change
With mucous
With billirubin
How is the urgent prophylaxis of scarlet fever conducted?
By vaccination
Isolation of children, who had contact with a patient
Using of vaccination
Disinfection
Non-admission of contact with carrier of B-streptococcus
How is the urgent prophylaxis of scarlet fever conducted?
By vaccination
Isolation of children, who had contact with a patient
Using of vaccination
Disinfection
E.
445.
A.
B.
C. *
D.
E.
446.
A.
B.
C. *
D.
E.
447.
A.
B.
C. *
D.
E.
448.
A.
B.
C. *
D.
E.
449.
A.
B.
C. *
D.
E.
450.
A.
B.
C. *
D.
E.
Non-admission of contact with carrier of B-streptococcus
In 1 liter of Trisalt solution, the concentration of potassium chloride is:
3 g/l
1.5 g/l
1.0 g/l
2 g/l
2.5 g/l
In a child with the clinical display of acute respiratory viral infection observed generalized
lymphadenopathy, one-sided conjunctivitis, increase of liver and spleen. What will be the most
credible diagnosis?
Infectious mononucleosis
Leptospirosis
Adenoviral infection
Influenza
Meningococcal infection
In a child with the clinical display of acute respiratory viral infection observed generalized
lymphadenopathy, one-sided conjunctivitis, increase of liver and spleen. What will be the most
credible diagnosis?
Infectious mononucleosis
Leptospirosis
Adenoviral infection
Influenza
Meningococcal infection
In a different places of settlement found out a few cases of cholera. Who in the focus of cholera was
send in an insulator?
Carriers
Persons contact with the patient
Patients with cholera
Persons with dysfunction of alimentary tract
Persons with hyperthermia
In a patient of 16 years old, the disease began gradually, from the catarrhal syndrome. For 2-3 days
the temperature of body increase till 38,5 °C, cold, severe cough with the negligible quantity of
mucous sputum, «souring» eyes. Peripheral lymph nodes are soft, painless and some enlarged.
Mucous of pharynx is hyperemic, granules on posterior part of pharynx. General state is satisfactory.
What is the drug of choice?
Ascorbic acid
Aspirin
Desoxyribonucleas
Remantadin
Aminocapronic acid
In a patient with ARVI fever develops to 40,1 °C, breathing frequency 40/min. What measures are
the most effective in treatment of such complication.
Minimising of body temperature
Keep patient on artificial lung ventilation
Oxygen inhalation
Infusion therapy
Antibiotic therapy|
451.
A.
B.
C. *
D.
E.
452.
A.
B.
C.
D. *
E.
453.
A. *
B.
C.
D.
E.
454.
A.
B. *
C.
D.
E.
455.
A.
B.
C.
D.
E. *
456.
A.
B.
C. *
D.
E.
457.
A.
B.
C. *
In a patient with ARVI fever develops to 40,1 °C, breathing frequency 40/min. What measures are
the most effective in treatment of such complication.
Minimising of body temperature
Keep patient on artificial lung ventilation
Oxygen inhalation
Infusion therapy
Antibiotic therapy|
In a settlement was found out a few cases of cholera. Who must be insulated?
Persons with disfunction of intestine
Patients with cholera
Carriers
Persons contact with the sick patient
Persons with hyperthermia
In girl V., 1 year old, appeared the thump of nose, dry cough, body temperature rose till 37,5 ?C.
Next day cough become attack like with the excretion of small amount of viscid sputum. Noisy
breathing. Sharply expressed expiratory dyspnea, breating rate 40 times/minute. During examination:
acrocyanosis and emphysematous thorax, at lungs dissipated dry and single moist rales. Tear of
frenulum of tongue. What will be the preliminary diagnosis?
Parainfluenza
Pneumonia
Influenza
Respiratory-syncytial infection
Whooping-cough
In the blood analysis at an enterorrhagia:
Leukocytosis and hyperhemoglobinemia
Coagulation failure
Leukocytosis
Normocytosis
Hyperhemoglobinemia
In the break out of cholera it is necessary to carry out such measures, except:
Hyperchlorination of drinking water
An active discovery of patients by rounds
Obligatory hospitalization, inspection and treatment of patients and vibrio tests
Revealing and isolation of contact persons
Vaccine prophylaxis
In the different places of settlement found out a few cases of disease of cholera. Who from the
contacts of cholera patient is sent in an insulator?
Vibrio positive
Patients with cholera
Contact with the patient persons
Persons with dysfunction of intestine
Persons with high temperature
In the different places of settlement it is found out a few cases of cholera. Who from such place is
directed to an insulator?
Patients with a cholera
Transmitters
Persons who had contact with the patient
D.
E.
458.
A.
B.
C. *
D.
E.
459.
A. *
B.
C.
D.
E.
460.
A.
B.
C.
D. *
E.
461.
A. *
B.
C.
D.
E.
462.
A.
B.
C. *
D.
E.
463.
A.
B.
C.
D. *
E.
Persons with dysfunction of gastro-intestinal tract
Persons who left the place on infection
Method of etiotropic therapy of cholera is.
Glucocorticoids
Antiviral
Antibiotics
Rehydration
Vaccine
More often the dehydrationous shock develops at:
Acute intestinal diseases
Respiratory diseases
Blood infections
Diseases of investments
Diseases of never system
Normal potassium concentration in blood plasma:
1,5-2,0 mmol/l
2,0 mmol/l
2,5 mmol/l
3,5-5,5 mmol/l
4,5 mmol/l
Normal sodium concentration in blood plasma:
135-150 mmol/l
125 mmol/l
170 mmol/l
110 mmol/l
90 mmol/l
Patient 22 years old, has increase temperature of body till 37,8 °C. Treated under the supervision of
district doctor with a diagnosis of influenza. On the 5th day of illness temperature remained the same;
it began difficultly in opening eyes. On examination – edema on face, expressed conjunctivitis with
film stratifications. Mucous pharynx is heperemia, on the back wall of gullet considerable graininess.
Lymph nodes are enlarged in neck. The general state of patient is satisfactory. This disease is related
to cold. What disease you suspect?
Leptospirosis
Infectious mononucleosis
Adenoviral infection
Allergic dermatitis
Meningococcal infection
Patient A., 28 years old, hospitalized with a previous diagnosis of flu. On the 5th day of illness, rash
appeared on the trunk and internal surfaces of extremities. Temperature 41.5 °C, hyperemia of sclera,
tremor of tongue, tachycardia, spleenomegaly, excitation. What is the most possible diagnosis?
Measles
Meningococcal infection
Leptospirosis
Epidemic typhus
Typhoid
464.
A.
B.
C.
D. *
E.
465.
A.
B.
C. *
D.
E.
466.
A. *
B.
C.
D.
E.
467.
A.
B. *
C.
D.
E.
468.
A.
B.
C.
D. *
E.
Patient B., 20 years old, complains about severe headache in temples and orbits, dull ache in the
trunk, dry cough. Temperature of the body 39.6 °C. Inflammatory changes of mucous membrane of
oropharynx. Normal breathing in the lungs. What is the most credible diagnosis?
Pneumonia
Parainfluenza
Respiratory micoplasma
Flu
Meningococcal infection
Patient L., 18 years old is sick with fever till 38 °C which proceeds 5 days. he has moderate dry
cough, common cold, badly opens eyes. On examination –edema on face, expressed conjunctivitis
with film raids. Mucous of pharynx is hyperemic, posterior wall of pharynx is grainy. Internal organs
are without pathology. What form of disease does the described picture correspond to?
Viral conjunctivitis
Allergic dermatitis
Adenoviral infection
Influenza
Rhinoviral infection
Patient M., 11 years old, complains on general weakness, cough, at night suddenly temperature rose
till 39,5 ?C, appeared restlessness, barking cough, noisy whistling breathing with drowing in supraand subclavicular cavities, intercostal spaces. He was in contact with the patient acute respiratory
viral infection. What should recommend him the first line?
Prednisolon, hot foot-baths
Seduxsen, euphylin
Euphylin, vitamin C
Antibiotics, dimedrol
Astmopen, diazolin
Patient P., 14 years old, is hospitalized in the infectious department in the severe condition.
Complains on expressed headache, mainly in frontal and temporal regions, supercilliary arcs, origin
of vomiting appear in condition of severe pain, pains by moving the eyeballs, in muscles and joints.
Objectively: patient is excited, body temperature-39 ?C. BP-100/60 mmHg. Bradycardia was
replaced by tachycardia. Appeared tonic cramps. Doubtful meningeal signs. From anamnesis it is
clear that his brother has flu at home. What will be your diagnosis?
Influenza, typical flow
Influenza with the phenomena of edema of brain
Respiratory-syncytial infection
Parainfluenza
Adenoviral infection
Patient R., 16 years old, hospitalized for 5-day illness with complaints of moderate headache in
fronto-temporal region, laid nose, sore throat, pain in the left eye, rise in temperature to 38.1-38.5 °C.
General condition is satisfactory. Shortness of nasal breath, mucous discharging from the nose,
hyperemia of face, enlargment of the neck and submaxillary lymph glands, left foamy conjunctivitis.
What is preliminary diagnosis?
Influenza
Infectious mononucleosis
Enteroviral infection
Adenoviral infection
Influenza
469.
A.
B. *
C.
D.
E.
470.
A.
B.
C.
D.
E. *
471.
A.
B. *
C.
D.
E.
472.
A.
B. *
C.
D.
E.
473.
A.
B.
C.
D.
E. *
474.
A.
B.
Patient R., 26 years old, became ill sharply: temperature 39,5 °C, severe headache, mainly in frontal
and temporal an area, pains in muscles and joints. Examined on the 2th days of illness: state of
middle weight, skin clean, dry. Moderate hyperemia with cyanosis, pulse 120/min., rhythmic.
Cardiac activity rhythmic, tones are muffled, in lights of the vesicular breathing. Stomach is without
peristalsis. What is the preparation of choice for treatment of this patient?
Aspirin
Remalol
Ampicillin||
Ascorbic acid
Ribonuclease
Patient with cholera has bradycardia, low blood pressure, weakness. What is most important factor in
the given clinical situation?
Hypocalcemia
Hypopotassium
Hyponatremia
Hypernatremia
Hyperpotassium
Patient with cholera on a background treatment has signs of hyperkalemia. What solution must be
applied for futher rehydration therapy?
Neohemodez
Disalt
Chlosalt
Polyhybrid
Lactosalt
Sick M., 22 years old, complaints about increasing of body temperature to 39 оC, headache in frontal
area, pain in eyeballs, photophobia, pain in a muscles, dry cough. Became ill suddenly. The state is
heavy. Objectively face is hyperemic, injection of scleras. Pulse 96 per min, rhythmic. Tones of heart
are hypotonic. In the lungs – dissipated dry wheezes. Mucous membrane of oropharynx is hyperemic,
grainy, vessels are extended. menengial symptoms are not present. Analysis of blood: leukocytes
3?109/L, е 1 %, band neut. 6 %, seg. neut 51 %, lymphocytes 35 %, мonocytes 7 %. What is most
probable diagnosis?
Measles
Flu
Meningococcal disease
Epidemic typhus
Pneumonia
Sick, 52 years old, with complaints about pain in lumbar region, headache edema of chin. It is known
from anamnesis that the sick suffers from obesity of ІІ degree. Recently carried heavy neurological
stress and had flu. He has chronic bronchitis for 5 years, chronic gastritis for 8 years. Objectively:
Temperature of the body 38.2 °C, AP – 140/90 mm Hg. It is proposed the diagnosis of acute
glomerulonephritis. What transferred factors could be the reason of disease?
Neuro psycologic stress
Chronic bronchitis
Chronic gastritis
Obesity
Flu
The bowel perforation of the typhoid fever can appear:
On 1-5 weeks of disease
On 1 week of disease
C.
D. *
E.
475.
A. *
B.
C.
D.
E.
476.
A. *
B.
C.
D.
E.
477.
A.
B.
C. *
D.
E.
478.
A.
B.
C.
D.
E. *
479.
A.
B.
C.
D.
E. *
480.
A.
B.
C.
D. *
E.
481.
A.
B.
On 2 week of disease
On 3 week of disease
On 5 week of disease
The bowel perforation of the typhoid fever is accompanied by the onset of:
Protective muscle tension of a stomach
Absence of respiratory excursion of a abdominal wall
Pains in a stomach
Disappearances of hepatic dullness of percussion
All answers are right
The collapse, a severe intoxication of a typhoid fever develops:
On 1-2 week of disease
On 3 week of disease
On 4 week of disease
On 4-5 week of disease
On 5-6 week of disease
The diarrheic syndrome and vomiting are the reason:
An infectious-toxic shock
An anaphylactic shock
A dehydrationous shock
A hemorrhagic shock
All right
The essential therapy for cholera is.
Diet
Antibacterial preparations
Correction of dysbacteriosis
Desintoxication
Primary rehydration
The hypovolemic shock develops owing to fluid loss at:
A long-term fever
A bleeding (a hemorrhagic shock)
Vomiting and diarrheas
All answers are not true
All answers are true
The hypovolemic shock is:
І degree of dehydration
ІІ degree of dehydration
ІІІ degree of dehydration
ІV degree of dehydration
V degree of dehydration
The patient P., 14 years old, is suffering from flu. He is hospitalized in infectious dept. due to
worsening of his condition. He is conscious. A patient is suffocated. Pallor of skin covers with
cyanosis, breathing rate 50 times/minute. BP-80/55 mmHg, pulse 110 times /minute. Body
temperature-39,5 °C. Excretion of rose foamy sputum. On percussion of lungs there is tympanic
sound with dullness in lower part of lung .On auscultation there is moist rales in lower posterior part
of lungs. What complication of flu appeared in patient?
Bronchitis
Edema of brain
C. *
D.
E.
482.
A.
B.
C.
D.
E. *
483.
A.
B. *
C.
D.
E.
484.
A.
B.
C. *
D.
E.
485.
A.
B.
C. *
D.
E.
486.
A.
B. *
C.
D.
E.
487.
A.
B.
C.
D. *
E.
488.
A.
B.
C. *
Pneumonia
Edema of lungs
infectious-toxic shock
The peritonitis of the typhoid is accompanied by the onset of:
Stefanskyy's symptom
Mayo-Robson's symptom
Voskresensky symptom
Krymov's symptom
Guarding symptom
The subcompensated dehydrationous shock develops at:
A diastolic and systolic blood pressure boost
A decrease of the systolic blood pressure
A diastolic blood pressure decrease
A diastolic blood pressure boost
A systolic blood pressure boost
The typhoid fever enterorrhagia is characterised with:
A normal pulse
A bradycardia
A tachycardia
An alternating pulse
An asystole
To the district doctor a patient, complaints on abundant excretions from a nose, moderate headache,
hearing loss, perversion of taste. On examination – dry of skin, nose excoriation, in a pharynx – mild
hyperemia. Temperature of body is subfebrile. Pathological changes of internal organs are absent.
Which acute respiratory viral infection carries the patient?
Adenoviral infection
Parainfluenza
Rhinoviral infection
РC-infection
Influenza
Typhoid fever bleedings appears:
On 5-6 week of disease
On 3-4 week of disease
On 1-2 week of disease
On 2 week of disease
On 1 week of disease
Typhoid fever bleedings are accompanied with:
Body temperature decrease and pulse decrease
Body temperature increase and pulse increase
The temperature does not change
Body temperature decrease and pulse increase
Body temperature increase and pulse decrease
?Virus causing hemorrhagic cystitis, diarrhea and conjunctivitis:
RSV
Rhinovirus
Adenovirus
D.
E.
489.
A.
B.
C. *
D.
E.
490.
A. *
B.
C.
D.
E.
491.
A.
B.
C.
D.
E. *
492.
A.
B. *
C.
D.
E.
493.
A. *
B.
C.
D.
E.
494.
A.
B.
C. *
D.
E.
495.
A.
B.
C. *
D.
E.
496.
A.
Rotavirus
Flu
?Virus causing hemorrhagic cystitis, diarrhea and conjunctivitis:
RSV
Rhinovirus
Adenovirus
Rotavirus
Flu
What clinically active forms of cholera do you know?
Very rapid acute for the children and elderly persons
“Choleric typhoid”, acute subclinical, for the children and elderly persons
Dry, very rapid, “choleric typhoid”, subclinical for the children and elderly persons
Very rapid “choleric typhoid”, acute, subclinical, for the children and elderly persons
Very rapid, dry, subclinical, for the children and elderly persons
What from is the given measures during the secondary rehydration?
Determining degree of dehydration from clinical data
Amount of lost liquid, which was preceded at the time of hospitalization
Application of isotonic crystalloid solutions
Simultaneous introduction of liquid in a few vessels
Amount of liquid loss
What from the below mentioned preparations can be used for the treatment of primary rehydration?
Lactosalt
Disalt
Acesalt
Trisalt
Khlosalt
What from the below mentioned preparations, can be used for the treatment of primary rehydration?
Polyhybrid
Acesalt
Khlosalt
Kvartasalt
Lactosalt
What from the below mentioned preparations, can be used for the treatment of primary rehydration?
Acesalt
Trisalt
Cryoplasma
Khlosalt
Lactosalt
What is conduct specific passive immunnoprophylaxis of flu?
Living antenuated vaccine
Inactive parenteral vaccine
By an immunoprotein
Remantadin
Antibiotics of wide spectrum of action|
What is conduct specific passive immunnoprophylaxis of flu?
Living antenuated vaccine
B.
C. *
D.
E.
497.
A.
B.
C.
D. *
E.
498.
A.
B.
C.
D. *
E.
499.
A.
B. *
C.
D.
E.
500.
A.
B. *
C.
D.
E.
501.
A.
B.
C. *
D.
E.
502.
A.
B.
C.
D. *
E.
503.
A.
B.
C. *
D.
E.
Inactive parenteral vaccine
By an immunoprotein
Remantadin
Antibiotics of wide spectrum of action|
What laboratory and instrumental examinations are needed for confirming the diagnosis of flu?
Complete analysis of blood
-ray of organs of thoraxic cavity
Analysis sputum|
Determination of viruses by the method of immunofluorescence
Biochemical blood test
What laboratory and instrumental examinations are needed for confirming the diagnosis of flu?
Complete analysis of blood
-ray of organs of thoraxic cavity
Analysis sputum|
Determination of viruses by the method of immunofluorescence
Biochemical blood test
What level is necessary to reduce the temperature of patient’s body with hyperthermia?
39 °C
38 °C
37,5 °C
37 °C
38,5 °C
What level is necessary to reduce the temperature of patient’s body with hyperthermia?
39 °C
38 °C
37,5 °C
37 °C
38,5 °C
What solutions must be applied for compensatory rehydration in cholera?
Colloid
Hypertensive epitonic polyionic crystalloid
Isotonic polyionic crystalloid
Reosorbilakt
Isotonic solution of glucose
What time is it necessary to complete primary rehydration at dehydration shock?
3-5 hrs
0.5 hrs
2-3 hrs
1-1.5 hrs
4-6 hrs
What type of a diarrhoeia is typical for a salmonellosis?
Osmotic
Exudative
Secretory
Mixed
Toxic
504.
A.
B.
C.
D. *
E.
505.
A.
B.
C. *
D.
E.
506.
A.
B.
C. *
D.
E.
507.
A. *
B.
C.
D.
E.
508.
A.
B.
C.
D.
E. *
509.
A.
B.
C.
D. *
E.
510.
A. *
B.
C.
D.
E.
Which from the below is a complication of cholera?
Collapse
Infectious-toxic shock
Acute renal insufficiency
Dehydration shock
Status typhosis
Which salt solutions do not contain potassium?
Trisalt
Lactosalt
Disalt
Qudrosalt
Khlosalt
Who must be admitted in the hospital from the focus of cholera?
Carriers
Patients with cholera
Persons with disfunction of intestine
Contact persons
Persons with high temperature
With the purpose of specific prophylaxis of cholera is used:
Cholerogen-toxoid
Vaccine
Nitrofuranes
Immunoprotein
Antibiotics
What is the entrance gate at infectious mononucleosis?
Mucus of colon
Mucus of digestive highway
Epithelial mews of skin
Peyer‘s plate and follicles
Mucus of nazo-pharig
A boy 6 years was in the close touch with a patient with diphtheria. What treatment-prophylactic
measures need to be conducted, if vaccine anamnesis is unknown?
Introduction of AWDT vaccine
Antibacterial therapy
Introduction of ADT-м to the toxoid
Antibacterial therapy and double introduction of ADT toxoid
Antibacterial therapy and introduction of immunoprotein
A child 2 years carries a diphtherial croup. There was the stop of breathing on 2nd days of whey
therapy. What was the reason of asphyxia?
Mechanical obturation by tapes
Stenosis of larynx
Anaphylaxis shock
Whey illness
Paresis of respiratory musculature
511.
A. *
B.
C.
D.
E.
512.
A. *
B.
C.
D.
E.
513.
A. *
B.
C.
D.
E.
514.
A. *
B.
C.
D.
E.
515.
A. *
B.
C.
D.
E.
516.
A.
B.
C.
D. *
E.
517.
A diagnosis is “diphtheria of otopharynx” put to the 3-years-old child (not instiled through the refusal
of parents). Family lives in a 3-room apartment, a mother does not work, a father is the director of
factory. Specify the method of isolation of child.
Obligatory hospitalization
Hospitalization after clinical testimonies
Hospitalization after epidemiologys testimonies
Isolation in home terms
Does not need isolation
A new born child on 10th day of life became worse: Temperature 39.2 °C, no frequent vomiting,
generalized cramps, violations of consciousness, spastic paresis of left extremities. Month prior to his
birth herpes virus was present in the mother, which she did not treat. What disease is most possible?
Herpetic encephalitis
Meningococcal meningitis
Subarechnoid hemorrhage
Cerebral abscess
Violation of blood cerebral circulation
A patient came with complaints of sickly erosions on his penis. From anamnesis frequent appearance
of similar rashes is found out during a year. Objectively: on a balanus are the grouped blisters and
erosions, soft on palpation. What is your diagnose?
Recurrent herpes of ІІ type
Vulvar pemphigus
Primary syphyllis
Shankoform pyoderma
Recurrent herpes of ІІІ type
?A patient has herpetic meningitis. What preparation of specific therapy for viral neuro infection
should be given?
Acyclovir
Cefataxime
Ceftriaxone
Gentamycin
Furazolidon
A patient has herpetic meningitis. What preparation of specific therapy for viral neuro infection
should be given?
Acyclovir
Cefataxime
Ceftriaxone
Gentamycin
Furazolidon
A patient has herpetic meningitis. What preparation for specific therapy of viral neiroinfection would
you appoint?
Laziks
Cefotaksim
Ceftriakson
Acyclovir
Prednisolon
A pregnant woman, 27 years (pregnancy ІІ, 8-10 weeks.), temperature of body increased. At the
inspection on a TORCH-infection antibodies are found to the herpes virus, ІІ types of class IGM.
What we must recommend to pregnant women?
A. *
B.
C.
D.
E.
518.
A. *
B.
C.
D.
E.
519.
A.
B. *
C.
D.
E.
520.
A.
B.
C.
D. *
E.
521.
A. *
B.
C.
D.
E.
522.
A.
B.
C.
D.
E. *
523.
A. *
B.
To cut pregnancy
To prolong the supervision
Treatment with acyclovir
Symptomatic treatment
Appoint of alpha-fetoprotein
A sick 3 years old child came to the doctor with symptoms of the fever, languor, waiver of meal. A
boy is capricious, temperature of body 37.9 °C. On the mucus shell of soft palate, cheeks are single
vesicle hypersalivation. What is the diagnosis?
Herpetic stomatitis
Candidosis of oral cavity
Leucoplacia
Follicular tonsillitis
Lacunar tonsillitis
After the disease which was accompanied by the fever and pharyngalgias, there were an
odynophagia, dysarthria, weakness and violation of motions in hands and feet, hyporeflexia, violation
of sensitiveness in extremities to the polyneurotic type. What disease does it follow to think about
Neuropathy
of hypoglossus
above all things?
Diphtherial polyneuropathy
Neuropathy of glossopharyngeus nerve
Trunk encephalitis
Pseudobulbar syndrome
At a child 4 years on the third day of disease the widespread form of diphtheria of nasopharynx is
diagnosed. Preparation of specific therapy:
Macrolids per os
Penicillin i/m
Cortycosteroid
Antidiphterial serum i/v
Antitoxic therapy
At a child 6 years with a diphtherial widespread croup the first dose of antidiphterial serum makes:
40 AО
15 AО
20 AО
80 AО
60 AО
At a girl, 22 years old, severy form of diphtheria of otopharynx have happened. Specific treatment
begun only on a 5th day from the beginning of disease. What complication of diphtheria is potentially
dangerous?
Stenotic laryngotracheitis
Pneumotorax
Meningoencephalitis
Septicopyemia
Infectious-toxic shock
At a patient the dense darkly-grey raid covers tonsills is considerably megascopic and spreads for
their scopes. Mucus shell bloodshot accented cyanochroic, was considerably swollen. Immediate
medical measure:
Antidiphterial whey
Punction of peritonsillar space
C.
D.
E.
524.
A. *
B.
C.
D.
E.
525.
A.
B.
C.
D. *
E.
526.
A.
B.
C. *
D.
E.
527.
A.
B.
C.
D. *
E.
528.
A.
B.
C.
D. *
E.
529.
A. *
B.
C.
D.
E.
Section of peritonsillar space
Microscopic research of stroke from under tape
Bacteriologic examination of stroke from under pallatum
At a patient the dense darkly-grey raid covers tonsills is considerably megascopic and spreads for
their scopes. Mucus shell bloodshot accented cyanochroic, was considerably swollen. Immediate
medical measure:
Antidiphterial serum
Punction of peritonsillar space
Section of peritonsillar space
Microscopic research of stroke from under tape
Bacteriologic examination of stroke from under pallatum
At a patient the dense darkly-grey raid covers tonsils is considerably megascopic and spreads for
their scopes. Mucus shell bloodshot accented cyanochroic, was considerably swollen. Immediate
medical measure:
Ultraviolet irradiation of throat
Punction of peritonsillar space
Section of peritonsillar space
Introduction of antidiphterial serum
Compress on a neck
At a patient which carried diphtheria with an incessant pharyngalgia, disartria, weakness and loss of
motions; hyperreflexia, decline of sensitiveness on a polyneuritis type developed. Put a diagnose.
Hemorrhage in a brain
Viral encephalitis
Diphtheria polyneuropathy
Psevdobulbar syndrome
Bulbar form of lateral Amiotrophic sclerosis
At a patient, 17 years: angina. Temperature 38,2 °C, generilised lymphadenopathy (the first
multiplied neck lymphatic knots which are located along m. sternocleidomastoideus), small icterus,
hepatospleenomegaly. Previous diagnosis?
Tuberculosis of lymphatic knots
Bacterial quinsy
Diphtheria
Infectious mononucleosis
Megacaryoblastoma
At how many percents of grown man does present antibodies to the virus of simple herpes?
10-20 %
20-30 %
40-60 %
80-90 %
60-70 %
At junior nurse, who works in child’s infectious department, herpes simplex was found. What should
manager of department must do?
Create a quarantine in the department
To appoint an immunoprotein to the children
Discharge all children from the department
To appoint immunomodulators with a prophylactic purpose
To inspect a junior nurse on a staphylococcus
530.
A. *
B.
C.
D.
E.
531.
A.
B.
C.
D. *
E.
532.
A.
B.
C. *
D.
E.
533.
A. *
B.
C.
D.
E.
534.
A. *
B.
C.
D.
E.
535.
A.
B.
C.
D. *
E.
536.
A.
B.
C. *
D.
E.
At junior nurse, who works in child’s infectious department, herpes simplex was found. What should
manager of department must do?
Create a quarantine in the department
To appoint an immunoprotein to the children
Discharge all children from the department
To appoint immunomodulators with a prophylactic purpose
To inspect a junior nurse on a staphylococcus
At maintenance of call on a house a district pediatrician put to the sick 5 years old child diagnosis
“Acute lacunar tonsillitis”. Specify, who must carry out the laboratory inspection of patient and in
what terms.
Worker of SES upon receipt report
A district medical sister is at once after determination of diagnosis
Doctor pediatrician in 5 hours
Doctor pediatrician at once after determination of diagnosis
District medical sister on a next day
At sick L, 35 years old, a diagnosis is set is diphtheria of pharynx, noncommunicative form. What
first dose of antitoxic antidiphtheria whey is it necessary to appoint?
120 thousand of AО
80 thousand of AО
30 thousand of AО
50 thousand of AО
150 thousand of AО
Before revaccination from diphtheria of adult persons, they are recommended:
To explore an immune type
To use antibiotics
To use antihistamines
5 years after last revaccination
10 years after last revaccination
Before revaccination from diphtheria of adult persons, they are recommended:
To explore an immune type
To use antibiotics
To use antihistamines
5 years after last revaccination
10 years after last revaccination
Complication of diphtheria of larynx:
Myocarditis
Paresis of auditory nerve
Nephrosonephritis
Croup
Poliomyelitis
Complications of 4-5th week of diphtheria:
Encephalitis
Bulbar disorders, pancreatitis, hepatitis
Poliomyelitis, myocarditis
Nephrosonephritis
Stenotic laryngotracheitis
537.
A.
B.
C.
D.
E. *
538.
A.
B. *
C.
D.
E.
539.
A.
B. *
C.
D.
E.
540.
A.
B. *
C.
D.
E.
541.
A. *
B.
C.
D.
E.
542.
A.
B.
C. *
D.
E.
543.
A.
B.
C. *
D.
E.
544.
A.
B. *
C.
Complications which often develop on the first week of diphtheria of otopharynx:
Poliomyelitis
Asphyxia
Insufficiency of glandulars
hepatospleenomegaly
Paresis of soft palate
Corynebacterium diphtheria:
Contain endotoxin only
Exotoxin products
Exotoxin does not product
An enterotoxin products
Myelotoxin products
Diphtheria planned vaccination begin in:
In first days after birth of child
In 3 month age
In 6-month age
In 1 year
In 6 years
Diphtheria planned vaccination begin in:
In first days after birth of child
In 3 month age
In 6-month age
In 1 year
In 6 years
Early complications of diphtheria of otopharynx:
Paresis of soft palate
Pneumonia
Asphyxia
Croup
Poliomyelitis
Especially high title of ant diptherial antitoxic antibodies testifies to:
Recovering
Acute period of diphtheria
Bacteriocarriering
Forming of immunity to diphtheria
About nothing does not testify
Especially high titre of ant diptherial antitoxic antibodies testifies to:
Recovering
Acute period of diphtheria
Bacteriocarriering
Forming of immunity to diphtheria
About nothing does not testify
For what disease are characterize changes in blood (presence of atypical mononucleares)?
Flu
Kissing disease
Measles
D.
E.
545.
A.
B. *
C.
D.
E.
546.
A.
B. *
C.
D.
E.
547.
A.
B.
C.
D.
E. *
548.
A.
B.
C.
D. *
E.
549.
A.
B.
C. *
D.
E.
550.
A.
B.
C. *
D.
E.
551.
A. *
B.
C.
AIDS
Diphtheria
For what disease characterize changes in a blood (presence of lymphomonocytes and a typical
mononuclears)?
Flu
Infectious mononucleosis
Measels
AIDS
Diphtheria
For what disease characterize changes in a blood (presence of lymphomonocytes and a typical
mononuclears)?
Flu
Infectious mononucleosis
Measels
AIDS
Diphtheria
How mach are exist subfamilies of herpes-viruses?
2
4
5
6
3
How mach types of herpes-viruses do you know?
2
4
6
8
10
In an epidemic cell rationally to organize verification of the state of immunity. The Use of RUHA
allows to find out persons unimmune to diphtheria during a few hours. What minimum protective
titre?
1:10
1:20
1:40
1:80
1:160
In preschool is case of disease on diphtheria. What prophylactic measures must be conducted above
all things?
Urgent hospitalization
Urgent vaccination
Quarantines measures
Urgent by chemical prophylactic antibiotics
Introduction of antidiphterial whey
In preschool the registered case of diphtheria. What from the measures adopted below does not
conduct to the contact children?
Introduction of antidiphterial whey
Non-permanent is stroke from a pharynx and nose for the bacteriologic examination
Daily is supervision during 7 days
D.
E.
552.
A.
B.
C.
D.
E. *
553.
A.
B.
C.
D.
E. *
554.
A. *
B.
C.
D.
E.
555.
A. *
B.
C.
D.
E.
556.
A.
B.
C. *
D.
E.
557.
A. *
B.
C.
D.
E.
558.
A. *
B.
C.
D.
Determination of titres of specific antibodies
At the repeated cases of disease is extraordinary revaccination diphtheria
In what age are infected by primary herpes more frequent?
55-65 years
5-10 years
12-18 years
to 6 months
6 months – 5 years
Is a vaccination conducted at a kissing disease?
Ribosom vaccine
Alive vaccine
Dead vaccine
Chemical vaccine
On the stage of
Name the most reliable of kissing disease?
Became healthy
Death
Chronic form
Hematological violations
Changes in the nervous system
Patient C., 60 years old during one year has 4th relapse of Herpes zoster. Recommended treatment
and relapses prophylaxis?
Valcyclovir
Acyclovir
Herpevir
Proteflazid
Cycloferon
Patient, 24 years old, with diagnosis dyphtheria was admitted to the infectious disease department.
What remedy is most effective for treatment and should be used immediately?
Antibiotics
Oxygenotherapy
Antitoxic antidyphtherial serum
Antipyretic drugs
Sulfanilamides
Patient, 35 years was hospitalized with diagnosis localized diphtheria of pharynx. What is the first
dose of antitoxic antidyphtherial serum?
30 000 AU
50 000 AU
80 000 AU
120 000 AU
150 000 AU
Rules hospitalizations of patients with a kissing disease?
In a room for the patients with infections of respiratory tracts
Patients are not hospitalized
In a separate chamber
In a chamber for the patients with infections of external covers
E.
559.
A.
B.
C.
D. *
E.
560.
A.
B. *
C.
D.
E.
561.
A. *
B.
C.
D.
E.
562.
A.
B. *
C.
D.
E.
563.
A.
B.
C. *
D.
E.
564.
A. *
B.
C.
D.
In a chamber for the patients with intestinal infections
Specify the correct method of introduction of whey after the method of Bezredko:
1,0 ml of divorced 1:100 hypodermic – through 30 min. 0,1 ml of undivorced hypodermic – through
30 min. all dose of intramuscle
0,1 ml of divorced 1:1 000 endermic – through 30 min. 0,1 ml of divorced 1:10 hypodermic –
through 30 min. all dose of intramuscle
0,1 ml of undivorced endermic – through 30 min. 0,1 ml hypodermic – through 30 min. all dose of
intramuscle
0,1 ml of divorced 1:100 endermic – through 30 min. 0,1 ml of undivorced hypodermic – through 30
min. all dose of intramuscle
1,0 ml of divorced 1:10 hypodermic – through 30 min. 0,1 ml of undivorced hypodermic – through
30 min. all dose of intramuscle
Specify the correct method of serum introduction after the Bezredko method:
1,0 ml of divorced 1:100 hypodermic – through 30 min. 0,1 ml of undivorced hypodermic – through
30 min. all dose of intramuscle
0,1 ml of divorced 1:1 000 endermic – through 30 min. 0,1 ml of divorced 1:10 hypodermic –
through 30 min. all dose of intramuscle
0,1 ml of undivorced endermic – through 30 min. 0,1 ml hypodermic – through 30 min. all dose of
intramuscle
0,1 ml of divorced 1:100 endermic – through 30 min. 0,1 ml of undivorced hypodermic – through 30
min. all dose of intramuscle
1,0 ml of divorced 1:10 hypodermic – through 30 min. 0,1 ml of undivorced hypodermic – through
30 min. all dose of intramuscle
The duration of latent period at a kissing disease are?
25-50 days
3-6 days
1-4 weeks
From a few hours to 3 days
From a few days to 1-2 months
The exciter of diphtheria is:
Virus of Epshtein-Barr
Bacillaof Leffler
Corynebacteria ulcerans
Fusiform stick
Corynebacteria xerosis
The otolaryngologist during the review of patient marked hyperemia, considerable edema of tonsills
with the grey raid on them. During the microscopy of raid it was found out sticks located under a
corner to each other. What disease does it follow to think about?
Scarlet fever
Streptococcus quinsy
Diphtheria
Vensan tonsillitis
Staphylococcus quinsy
The source of infection at diphtheria is:
Sick people and bacillicarriers
Sick agricultural animals
Rodents
Mosquito
E.
565.
A. *
B.
C.
D.
E.
566.
A. *
B.
C.
D.
E.
567.
A. *
B.
C.
D.
E.
568.
A.
B.
C.
D. *
E.
569.
A.
B.
C.
D. *
E.
570.
A. *
B.
C.
D.
E.
571.
A.
B.
C. *
D.
E.
572.
A.
B. *
Aerosol of saliva and epipharyngeal mucous of patients
The source of infection at diphtheria is:
Sick people and carriers
Sick agricultural animals
Rodents
Mosquitoes
Aerosol of saliva and epipharyngeal mucous of patients
The source of infection at infectious mononucleosis is:
Sick people and carriers
Sick agricultural animals
Rodents
Mosquitoes
Aerosol of saliva and epipharyngeal mucous of patients
What additional inspections must be conducted to the patient with infectious mononucleosis?
IFA on HIV-infection, bacteriology inspection on diphtheria
IFA on HIV-infection, bacteriology inspection on a rabbit-fever
Bacteriology inspection on diphtheria and typhoid
Reaction of Burne and Rihth-Heddlson
Reaction of Paul-Bunnel and punction of lymphatic knot
What additional inspections must be conducted to the patient with a kissing disease?
Reaction of Burne and Rayt
ELISA test on AID, bacteriological examination on a rabbit-fever.
Bacteriological examination on diphtheria and typhoid
ELISA test on AID, bacteriological examination on diphtheria
Reaction of Paul-Bunnel and punction of lymphatic no
What additional test should hold for the patient with infectious mononucleosis?
Burne and Wright-Hadlson‘s reactions
ELISA-test, bacteriological test for tularemia
Bacteriological test for diphtheria and typhoid fever
ELISA-test, bacteriological test for diphtheria
Paul-Burne reaction and lymph node puncture
What are the possible ways of transmission of herpes-viruses?
Contact, air, sexual, vertical
Contact, sexual, vertical
Contact, air, vertical
Contact, air, sexual
Air, sexual, vertical
?What are the rules of hospitalization of patients with infectious mononucleosis?
Patients are not hospitalized
In a chamber for the infections of respiratory tracts
In a separate chamber
In a chamber for the infections of external covers
In a chamber for intestinal infections
What characteristic signs of raid at diphtheria?
One-sided, grey-white, on-the-spot crateriform ulcers
grey-white, dense with clear edges and brilliant surface
C.
D.
E.
573.
A. *
B.
C.
D.
E.
574.
A.
B.
C.
D. *
E.
575.
A.
B.
C. *
D.
E.
576.
A.
B.
C.
D.
E. *
577.
A.
B.
C.
D.
E. *
578.
A.
B. *
C.
D.
E.
579.
A. *
B.
C.
Yellow-white, fragile, perilacunar is located
One-sided, yellow-white, in lacunas
White, fragile, is easily taken off by a spatula
What complication has developed in patient with diphtheria of mouth pellicle severe form was
diagnosed. On the 6th day of disease when pain in the heart region, palpitation were appeared. Pulse
– 120 per 1 min, systolic noise on apex of heart. On ECG is incomplete blockade of left leg of Giss
bunch?
Early infectious-toxic myocarditis
Myocardial dystrophy
Heart attack of myocardium
Acute cardio-vessel insufficiency
Stenosis of mitral valve
What complications do happen at a kissing disease?
Insult
Autoimmune diseases
Contractures
Break of spleens
Cirrhosis
What complications more often develops during 4-5th week of diphtheria:
Encephalitis
Bulbar disorders, pancreatitis, hepatitis
Poliomyelitis, myocarditis
Nephrosonephritis
Stenotic laryngotracheitis
What complications more often develops during the first week of diphtheria of otopharynx:
Poliomyelitis
Asphyxia
Paratonsillitis
Hepatospleenomegaly
Paresis of soft palate
What disease can the acute second tonsillitis be at?
Lupus
Diphtheria
Rheumatism
Tuberculosis
Typhoid
What disease is by the herpes-virus of 1th type?
Genital herpes
L herpes
Syndrome of chronic fatigue
Sarcoma of Kaposi
Cytomegalovirus infection
What disease is by the herpes-virus of 2 type?
Genital herpes
L herpes
Syndrome of chronic fatigue
D.
E.
580.
A.
B.
C.
D. *
E.
581.
A.
B.
C.
D. *
E.
582.
A.
B.
C.
D.
E. *
583.
A.
B. *
C.
D.
E.
584.
A.
B. *
C.
D.
E.
585.
A.
B.
C. *
D.
E.
586.
A.
B.
C.
D. *
E.
587.
A. *
Sarcoma of Kaposi
Cytomegalovirus infection
What disease is by the herpes-virus of 3 type?
Genital herpes
L herpes
Syndrome of chronic fatigue
Herpes zoster
Cytomegalovirus infection
What disease is by the herpes-virus of 3 type?
Genital herpes
L herpes
Syndrome of chronic fatigue
Chicken pox
Cytomegalovirus infection
What disease is by the herpes-virus of 4 type?
Genital herpes
L herpes
Syndrome of chronic fatigue
Chicken pox
Cytomegalovirus infection
What disease is by the herpes-virus of 5 type?
Genital herpes
Eczema of new-born
Syndrome of chronic fatigue
Sarcoma of Kaposi
Epshtein-Barr‘s infection
What disease is by the herpes-virus of 6 type?
Genital herpes
Eczema of new-born
Syndrome of chronic fatigue
Sarcoma of Kaposi
Epshtein-Barr‘s infection
What disease is by the herpes-virus of 7 type?
Genital herpes
Eczema of new-born
Syndrome of chronic fatigue
Sarcoma of Kaposi
Epshtein-Barr‘s infection
What disease is by the herpes-virus of 8 type?
Genital herpes
Eczema of new-born
Syndrome of chronic fatigue
Sarcoma of Kaposi
Epshtein-Barr‘s infection
What family does the exciter of kissing disease belong to?
Family of herpes virus
B.
C.
D.
E.
588.
A.
B. *
C.
D.
E.
589.
A.
B. *
C.
D.
E.
590.
A.
B.
C.
D. *
E.
591.
A.
B.
C.
D. *
E.
592.
A.
B.
C. *
D.
E.
593.
A.
B.
C. *
D.
E.
594.
A.
B. *
C.
D.
E.
Family of pox virus
Family of retro virus
Family of reo virus
Family of toga virus
What from the following symptoms are not characteristic of infectious mononucleosis?
Fever
Defeat of kidneys
Lymphadenopathy
Tonsillitis
Increasing of liver and spleen
What from the following symptoms are not characteristic of infectious mononucleosis?
Fever
Defeat of kidneys
Lymphadenopathy
Tonsillitis
Increasing of liver and spleen
What group of infections does infectious mononucleosis behave to?
Zoonoz
Sapronosis
Antropozoonoz
Antroponoz
Sapronoz+antroponoz
What group of infectious diseases by L. Gromashevsky classification diphtheria belong to?
External covers
Blood
Intestinal
Respiratory ways
Transmissive
What group of infectious diseases diphtheria belong to?
Sapronosis
Zoonosis
Anthroponosis
Zooanthroponosis
A group is not certain
What group of infectious diseases infectious mononucleosis belong to?
Sapronosis
Zoonosis
Anthroponosis
Zooanthroponosis
A group is not certain
What is characteristic signs of raid at diphtheria?
One-sided, grey-white, on-the-spot crateriform ulcers
Grey-white, dense with clear edges and brilliant surface
Yellow-white, fragile, perilacunar is located
One-sided, yellow-white, in lacunas
White, fragile, is easily taken off by a spatula
595.
A. *
B.
C.
D.
E.
596.
A.
B.
C. *
D.
E.
597.
A.
B.
C.
D. *
E.
598.
A.
B.
C.
D. *
E.
599.
A.
B.
C. *
D.
E.
600.
A. *
B.
C.
D.
E.
601.
A.
B.
C. *
D.
E.
602.
A.
B. *
C.
What is early complications of diphtheria of otopharynx:
Paresis of soft palate
Pneumonia
Asphyxia
Croup
Poliomyelitis
What is immediately investigation in suspicious of diphtheria:
Strokes with tonsills, nose or other areas for the exposure of diphtherial stick
IFA
Microscopy (painting for Neiser)
Haemoculture
RDHA with a diphtherial diagnosticum
What is main complication of diphtheria of larynx:
Myocarditis
Paresis of auditory nerve
Nephrosonephritis
Croup
Poliomyelitis
What is material for the bacteriologic examination in time to suspicion on diphtheria?
Excrement
Blood
Urine
Mucus from the area of defeat
Neurolymph
What is mechanism of transmission of Corynebacterium diphtheria?
Vertical
Transmissive
Air-drop
Contact
Parenteral
What is recommended treatment and relapses prophylaxis of Herpes zoster?
Valcyclovir
Acyclovir
Herpevir
Proteflazid
Cycloferon
What is seasonal character of diphtheria?
Spring-summer
Summer-autumn
Autumn-winter
Winter-spring
Spring-autumn
What is the mechanism of transmission of herpetic infection?
Fecal-oral
Air
Contact
D.
E.
603.
A.
B. *
C.
D.
E.
604.
A.
B.
C. *
D.
E.
605.
A.
B.
C. *
D.
E.
606.
A.
B. *
C.
D.
E.
607.
A.
B.
C. *
D.
E.
608.
A.
B.
C.
D. *
E.
609.
A.
B.
C.
D. *
E.
610.
A.
Vertical
Transmisiv
What is the exciter of diphtheria:
Virus of Epshtein-Barr
Leffler Bacillus
Corynebacteria ulcerans
Fusiform stick
Corynebacteria xerosis
What is the exciter of kissing disease:
Virus small pox
Virus of simple herpes
Virus of Epshteyn-Barr
Cytomegalovirus
Virus of flu
What is the most diagnostic method for infectious mononucleosis?
Common analysis of excrement
Common analysis of urine
Common blood test
Blood is on a drop
Stroke of blood
What is the properties of сorynebacterium diphtheria:
Contain endotoxin only
Exotoxin products
Exotoxin does not product
An enterotoxin products
Myelotoxin products
What is transmissive factors in diphtheria?
Blood
Water
Saliva
Urine
Exrements
What laboratory examination is compulsory to do for the patient with signs of tonsillit?
Isolation of hemolytic streptococcus from the throat mucosa
Biochemical blood analysis
X-ray examination
Smear from nose and pharynx
Immune-enzyme analysis
What material it’s necessary to take for bacteriologic examination in suspicion on diphtheria?
Excrement
Blood
Urine
Mucous
Neurolymph
What mechanism of transmission of Corynebacterium diphtheria?
Vertical
B.
C. *
D.
E.
611.
A.
B.
C. *
D.
E.
612.
A.
B. *
C.
D.
E.
613.
A.
B. *
C.
D.
E.
614.
A.
B.
C.
D. *
E.
615.
A.
B.
C.
D. *
E.
616.
A.
B.
C.
D. *
E.
617.
A. *
Transmissive
Air-drop
Contact
Parenteral
?What medical measures are primary in diphtheria of pharynx, widespread form?
ntroduction of non steroid and ant inflammatory drugs
ntroduction of antibiotics
ntroduction of ant diphtheria serum
ntroduction of glucocorticoids
isintoxication therapy
What symptom is not characterized for a kissing disease?
Generalized lymphadenopathy
Total flatulence
Tonsillitis
Hepatolienal syndrome
Rash
What symptoms do not characterize for infectious mononucleosis?
Increased of temperature
Defeat of
Lymphadenopathy
Tonsillitis
Increase of liver and spleen
What the most possible complication occurs during infectious mononucleosis?
Meningitis
autoimmune alopecia
encephalitis
Splenic rupture
Obstruction of respiratory tract
What the most possible complication occurs during infectious mononucleosis?
Meningitis
Autoimmune alopecia
Encephalitis
Splenic rupture
Obstruction of respiratory tract
What ways of transmission does characterize for infectious mononucleosis?
Alimentarniy
Transfuziv
Sexual
Air
Contact
A 24 y/o man .was seen by a doctor on the 2d day of illness with complaints about a fever up-to 38,1
°C, painful swallowing, malaise. On the skin of trunk and extremities, especially in natural folds,
point rash on hyperemic background was faund. The nasolabial triangle was white with no rash.
Enantema on a soft palate, and purulent exudates on tonsils were observed. A few enlarged and
painful submandibular lymph nodes were palpated. What is the most probable diagnosis?
Scarlet fever
B.
C.
D.
E.
618.
A.
B. *
C.
D.
E.
619.
A.
B.
C.
D. *
E.
620.
A.
B.
C. *
D.
E.
621.
A. *
B.
C.
D.
E.
622.
A. *
B.
C.
D.
E.
623.
A.
Rubella
Measles
Enteroviral infection
Flu
A 24.y/o man was seen by a doctor on the 2d day of illness with complaints about subfebrile
temperature, and insignificant catarrhal phenomena. On the skin of trunk and extremities
spotted-papular elements of rash without a tendency to congregate was observed. The mucous
membrane of oro-pharinx was not changed. A few enlarged and painful cervical lymph nodes were
found. What is the diagnosis?
Scarlet fever
Rubella
Measles
Enteroviral infection
Flu
A 24.y/o patient was seen by a doctor on the 2d day of illness with complaints about a fever up-to 38
°C, headache, and malaise. On the skin of hairy part of head, runk and extremities vesicular rash with
red hallow was found. Also some vesicular elements were found on the mucous membrane of
pharynx. Lymph nodes were not palpable. What is the most probable diagnosis?
Scarlet fever
Rubella
Measles
Chicken-pox
Herpetic infection
A 4 years old child complains about: cough, temperature of body 38,1 °C. Skin without rashes.
Conjunctiva hyperemic. Exanthema on skin. On mucous of cheeks there are points of hyperemia gum
blushs. In lungs difficult breathing. What is the most possible diagnosis?
Scarlet fever
Rubella
Measles
Enteroviral infection
Flu
A child 10 years old presents with temperature 38 °C, renitis, conjunctivitis, moist cough. On the
mucous membrane of cheeks, lips, gums there are greyish-white points, reminding a farina. What is
the diagnosis?
Measles
Adenoviral infection
URTI
Enteroviral infection
Infectious mononucleosis
A normal indices of impalpable fluid losses of the adult person with 70 kg body weight is:
1000 ml per a day
500 ml per a day
700 ml per a day
1500 ml per a day
2000 ml per a day
?A pathological state which develops owing to catastrophic reduction of a circulating fluid volume
and electrolytes loss is:
An infectious-toxic shock
B.
C. *
D.
E.
624.
A. *
B.
C.
D.
E.
625.
A.
B. *
C.
D.
E.
626.
A. *
B.
C.
D.
E.
627.
A.
B.
C. *
D.
E.
628.
A.
B. *
C.
D.
E.
629.
A.
B. *
C.
D.
E.
630.
An anaphylactic shock
A dehydrationous shock
A hemorrhagic shock
All right
A patient is sick with meningococcal meningitis. He take a massive dose of penicillin. 4 days
temperature of body 36,6-36,8 °C. Meningeal signs are negative. When is it possible to stop the
antibiotic therapy.
At a cytosis in a CSF 100 and less, lymphocytes prevail
After 10 days from the beginning antibiotic therapy
After 7 days from the beginning antibiotic therapy
At a cytosis 100 and less, neutrophil prevail
From 6 days from the beginning antibiotic
A patient with meningococcal meningitis gets penicillin during 7 days. The temperature of body is
normal 4 days. Meningeal signs are negative. When is it possible to stop the antibiotic therapy.
In default of leucocytosis displacement in blood
At a cytosis in a neurolymph 100 and less, lymphocyte prevail
At a cytosis in a neurolymph 100 and less, neutrophil prevail
At a cytosis in a neurolymph 150, lymphocyte prevail
At once immediately
A patient with meningococcal meningitis gets penicillin during 7 days. Last 4 days temperature of
body is normal. Meningeal signs are absent. When is it possible to abolish an antibiotic?
At cytosis in liquor 100 and less, lymphocytes prevails
At absence of leukocytosis and stab-nucleus shift in a blood
At cytosis in liquor 100 and more less, neutrophils prevails
At cytosis in liquor 150, lymphocytes prevails
At once
A patient’s temperature is 40 °C. There are olso deep and unproductive cough, photophobia, face
puffiness whitish points on the mucous membrane of cheeks opposite molar teeth. What is the most
possible diagnosis?
Tuberculosis
Meningococcemia
Measles
Enteroviral infection
Staphylococcus sepsis
All are the clinical signs of measles except:
Acute beginning of high fever
Icterus
Maculo-papular rash
Sequential appearance of rash
Scaling
All are the clinical signs of measles EXEPT:
Acute beginning of high fever
Icterus
Maculo-papular rash
Sequential appearance of rash
Scaling
All are the clinical signs of measles EXEPT:
A.
B. *
C.
D.
E.
631.
A.
B.
C. *
D.
E.
632.
A.
B.
C.
D.
E. *
633.
A.
B.
C.
D.
E. *
634.
A. *
B.
C.
D.
E.
635.
A.
B.
C.
D.
E. *
636.
A. *
B.
C.
D.
E.
637.
A. *
B.
C.
Acute beginning of high fever
Icterus
Maculo-papular rash
Sequential appearance of rash
Scaling
Among the students of school 2 cases of generalized form of meningococcal infection are registered.
What preparation does it follow to enter to the contact persons with the purpose of urgent
prophylaxis?
Normal
immune globulin
Leukocytic interferon
Meningococcal vaccine
Meningococcal anatoxin
Bacteriophage
And. 5 days after the isolation of the last patient
11 days
21 day
10 days
No need for quarantine
5 days after isolation of the last child
And. 5 days after the isolation of the last patient
11 days
21 day
10 days
No need for quarantine
5 days after isolation of the last child
Can the symptoms of an acute appendicitis be the complications of typhoid fever:
Yes
No
Not always
Often
May be
Compensated dehydrationous shock develops:
Rise level of toxins
At a decrease of the systolic blood pressure
At a hyperthermia
At hypohemoglobinemia
Because of the haemodynamics changes absence in peace
Enterorrhagia feces:
Melena
Fetid
Does not change
With mucous
With billirubin
Etiology agent of meningitis is:
Neisseria meningitides
Entamoeba histolytica
Vibro cholerae
D.
E.
638.
A. *
B.
C.
D.
E.
639.
A.
B.
C. *
D.
E.
640.
A.
B.
C. *
D.
E.
641.
A.
B.
C. *
D.
E.
642.
A.
B.
C. *
D.
E.
643.
A.
B.
C. *
D.
E.
644.
A.
B.
C.
D.
E. *
645.
A.
Clostridium botulinum
Campylobacter pylori
?Etiology agent of meningitis is:
Neisseria meningitides
Entamoeba histolytica
Vibrio cholerae
Clostridium botulinum
Campylobacter pylori
For how long a patient with complicated form of measles should be isolated:
For 4 days from the beginning of rash
For 7 days from the beginning of rash
For 10 days from the beginning of rash
For 17 days from the beginning
For 20 days from the beginning of illness
For how long a patient with complicated form of measles should be isolated:
For 4 days from the beginning of rash
For 7 days from the beginning of rash
For 10 days from the beginning of rash
For 17 days from the beginning
For 20 days from the beginning of illness
For how long a patient with complicated form of measles should be isolated:
For 4 days from the beginning of rash
For 7 days from the beginning of rash
For 10 days from the beginning of rash
For 17 days from the beginning
For 20 days from the beginning of illness
For the treatment of acidosis at meningococcal meningitis is better to use.
10-20 % glucose solution
10 % chloride solution
4 % sodium bicarbonate solution
Albumen
Concentrated dry plasma
For the treatment of acidosis at meningococcal meningitis is better to use.
10-20 % glucose solution
10 % chloride solution
4 % sodium bicarbonate solution
Albumen
Concentrated dry plasma
How is it possible to specify the diagnosis of meningococcal meningitis.
Meningitis is primary
Presence of a lot of cells in the CSF
Presence of gram-negative diplococcus in CSF
Meningococes from the throat
All the above
How is it possible to specify the diagnosis of meningococcal meningitis.
Meningitis is primary
B.
C.
D.
E. *
646.
A. *
B.
C.
D.
E.
647.
A.
B. *
C.
D.
E.
648.
A.
B. *
C.
D.
E.
649.
A.
B. *
C.
D.
E.
650.
A.
B. *
C.
D.
E.
651.
A.
B. *
C.
D.
E.
652.
A.
B. *
Presence of a lot of cells in the CSF
Presence of gram-negative diplococcus in CSF
Meningococes from the throat
All the above
In patients with intensive head acke, nausea, pain in a neck and lumbar area, expressed meningeal
symptoms; light, tactile, pain hyperesthesia what method of inspection is most informing?
Lumbar puncture
Computer tomography
Electroencephalography
Transcranial dopplerography
Echoencephalography
In the blood analysis at an enterorrhagia:
Leukocytosis and hyperhemoglobinemia
Coagulation failure
Leukocytosis
Normocytosis
Hyperhemoglobinemia
In the kindergarden a child had a meningococcal infection. She was immediately hospitalized. After
clinical convalescence in child sowed meningococcus. To which category can the carrier (child) of
pathogen belongs?
Convalescent, chronic
Convalescent, acute
Healthy
Immune in vaccinated
Immune in those, that had infection
In what daily interval should the dose of benzylpenicillin at meningococcal meningitis administered.
2 hrs
4 hrs
6 hrs
5 hrs
8 hrs
In what daily interval should the dose of benzylpenicillin at meningococcal meningitis administered.
2 hrs
4 hrs
6 hrs
5 hrs
8 hrs
In what dose should| benzyl penicillin be administered at meningococcal meningitis?
From a calculation 100-300 thousands unit on 1 kg of mass of body on days
From a calculation 200-500 thousands unit on 1 kg of mass of body on days
From a calculation 500-700 thousands unit on 1 kg of mass of body on days
From a calculation 700-900 thousands unit on 1 kg of mass of body on days
Regardless of mass of body
In what dose should| benzyl penicillin be administered at meningococcal meningitis?
From a calculation 100-300 thousands unit on 1 kg of mass of body on days
From a calculation 200-500 thousands unit on 1 kg of mass of body on days
C.
D.
E.
653.
A.
B. *
C.
D.
E.
654.
A.
B. *
C.
D.
E.
655.
A.
B. *
C.
D.
E.
656.
A.
B.
C.
D.
E. *
657.
A.
B.
C.
D.
E. *
658.
A.
B.
C.
D.
E.
659.
From a calculation 500-700 thousands unit on 1 kg of mass of body on days
From a calculation 700-900 thousands unit on 1 kg of mass of body on days
Regardless of mass of body
Measures of urgent prophylaxis for unvaccinated children who have never been ill with measles in
case of exposure to an ill with measles
Separation from the source
Vaccination
Administration of antibiotics
Disinfection
Does not exist
Measures of urgent prophylaxis for unvaccinated children who have never been ill with measles in
case of exposure to an ill with measles:
Separation from the source
Vaccination
Administration of antibiotics
Disinfection
Does not exist
Measures of urgent prophylaxis for unvaccinated children who have never been ill with measles in
case of exposure to an ill with measles
Separation from the source
Vaccination
Administration of antibiotics
Disinfection
Does not exist
Measures of urgent prophylaxis of measles for contacts which have never been ill, but were
vaccinated against measles
Separation from the source
Vaccination
Use of antibiotics
Use of immunoglobulin
No need to conduct
Measures of urgent prophylaxis of measles for contacts which have never been ill, but were
vaccinated against measles
Separation from the source
Vaccination
Use of antibiotics
Use of immunoglobulin
No need to conduct
Measures of urgent prophylaxis of measles for people who had been ill with measles, but never have
been vaccinated
Separation from the ill
Vaccination
Use of immunoglobulin
Use of antibiotics
No need to conduct
Measures of urgent prophylaxis of measles for people who had been ill with measles, but never have
been vaccinated:
A.
B.
C.
D.
E. *
660.
A.
B.
C.
D.
E.
661.
A.
B.
C. *
D.
E.
662.
A.
B.
C. *
D.
E.
663.
A.
B.
C.
D.
E.
664.
A.
B.
C.
D.
E. *
665.
A.
B.
C.
D.
E.
666.
A. *
B.
C.
Separation from the ill
Vaccination
Use of immunoglobulin
Use of antibiotics
No need to conduct
Measures of urgent prophylaxis of measles for people who had been ill with measles, but never have
been vaccinated
Separation from the ill
Vaccination
Use of immunoglobulin
Use of antibiotics
No need to conduct
Meningococemia and DIC-syndrome require above all things.
dministration of diuretics
Administration of analgesic
Administration of heparin
Administration of vitamins
Administration of antihistaminic preparations
Meningococсemia and DIC-syndrome require above all things.
Administration of diuretics
Administration of analgesic
Administration of heparin
Administration of vitamins
Administration of antihistaminic preparations
Methods of specific prophylaxis of scarlet fever:
Isolation of ill
Vaccination
Use of antibiotics
Disinfection
Does not exist
Methods of specific prophylaxis of scarlet fever:
Isolation of ill
Vaccination
Use of antibiotics
Disinfection
Does not exist
Methods of specific prophylaxis of scarlet fever:
Isolation of ill
Vaccination
Use of antibiotics
Disinfection
Does not exist
More often the dehydrationous shock develops at:
Acute intestinal diseases
Respiratory diseases
Blood infections
D.
E.
667.
A.
B.
C.
D. *
E.
668.
A. *
B.
C.
D.
E.
669.
A. *
B.
C.
D.
E.
670.
A.
B.
C. *
D.
E.
671.
A.
B.
C. *
D.
E.
Diseases of investments
Diseases of never system
Normal potassium concentration in blood plasma:
1,5-2,0 mmol/l
2,0 mmol/l
2,5 mmol/l
3,5-5,5 mmol/l
4,5 mmol/l
Normal sodium concentration in blood plasma:
135-150 mmol/l
125 mmol/l
170 mmol/l
110 mmol/l
90 mmol/l
Patient ?. 27 years old, was admited on the 4th day of illness with the diagnosis of viral URTI, and
allergic dermatitis”. The patient felt ill suddenly with increase of temperature up-to 38 °C, headache,
sore throat, and intensive cough. On the 3d day rash appeared on the skin of neck and face. Patient
was taking aspirin in order to decrease the temperature. Objectively: temperature of body 38,8 °C.
The face of patient looks puffy. Signs of conjunctivitis, and renitis were observed. On the skin of
neck face and chest there was intensive papular rash, without itch. Mucous membrane of oropharynx
was brightly hyperemic. Submandibular and frontal neck lymph nodes were enlarged. The liver and
spleen were not palpable. What is the diagnosis?
Measles
Allergic dermatitis
Infectious mononucleosis
Rubella
Scarlet fever
Patient A 24y/o. Appealed on the 5th day of illness with complaints about cough, malaise, fever up-to
38,1 °C. On face and behind the ears maculo-papular rash, appeared a day ago, and other areas of
skin are without rash. Conjunctiva is hyperemic. Enantema on a soft palate, on the mucous
membrane of cheeks are whitish points with the halo of hyperemia. There is the hard breathing in the
lungs. Most credible diagnosis is
Scarlet fever
German measles
Measles
Enteroviral infection
Flu
?Sick C., 8 years, appealed to the infectious hospital on the second day of disease with complaints
about a pharyngalgia at swallowing, increase of temperature. Objectively: temperature 38,6 °C, sharp
hyperemia of soft palate, tonsills, filling out, loose, on both there are festering stratifications which
are taken off by a spatula. Megascopic sickly submandibular lymphonoduses are palpated. Pulse –
114 per a min. Present roseol-papular pouring out on all body. Pastia‘s symptom is positive. It is
known from epidemogical anamnesis, that its comrade had alike symptoms. It is not found out the
change from the side of other organs. What previous diagnosis can be suspected?
Lacunar quinsy
Diphtheria of pharynx
Scarlet fever
Flu
Infectious mononucleosis
672.
A.
B.
C.
D.
E. *
673.
A.
B.
C.
D.
E. *
674.
A.
B.
C.
D.
E. *
675.
A.
B.
C.
D. *
E.
676.
A.
B.
C.
D. *
E.
677.
A.
B.
C.
D. *
E.
678.
A.
B.
C.
D. *
Sick C., 8 years, appealed to the infectious hospital on the second day of disease with complaints
about a pharyngalgia at swallowing, increase of temperature. Objectively: temperature of body – 38,6
°C, sharp hyperemia of soft palate, tonsills, filling out, loose, on both there are festering
stratifications which are taken off by a spatula. Palpated megascopic, sickly submandibular
lymphonoduses. Pulse – 114 shots per min. Present roseol-papular pouring out on all body. Pastia
symptom is positive. It is known from epid anamnesis, that at its comrade were alike symptomes. It is
not found out the change from the side of other organs. What previous diagnosis can be suspected?
lacunar quinsy
Diphtheria of pharynx
Infectious mononucleosis
Flu
Scarlet fever
Source of meningitis is:
Animals
Birds
Fish
Pediculus humanus
People
Source of meningitis is:
Animals
Birds
Fish
Pediculus humanus
People
Term of contagious period of patient diagnosed with uncomplicated form of measles
Until clinical recovery
After rash starts disappearing
Before appearance of rash
4 days from the beginning of rash
10 days from the beginning of illness
Term of contagious period of patient diagnosed with uncomplicated form of measles
Until clinical recovery
After rash starts disappearing
Before appearance of rash
4 days from the beginning of rash
10 days from the beginning of illness
Term of contagious period of patient diagnosed with uncomplicated form of measles
Until clinical recovery
After rash starts disappearing
Before appearance of rash
4 days from the beginning of rash
10 days from the beginning of illness
The bowel perforation of the typhoid fever can appear:
On 1-5 weeks of disease
On 1 week of disease
On 2 week of disease
On 3 week of disease
E.
679.
A. *
B.
C.
D.
E.
680.
A. *
B.
C.
D.
E.
681.
A.
B.
C. *
D.
E.
682.
A.
B.
C.
D.
E. *
683.
A.
B.
C.
D. *
E.
684.
A.
B.
C.
D.
E. *
685.
A.
B. *
C.
D.
E.
686.
A.
B.
On 5 week of disease
The bowel perforation of the typhoid fever is accompanied by the onset of:
Protective muscle tension of a stomach
Absence of respiratory excursion of a abdominal wall
Pains in a stomach
Disappearances of hepatic dullness of percussion
All answers are right
The collapse, a severe intoxication of a typhoid fever develops:
On 1-2 week of disease
On 3 week of disease
On 4 week of disease
On 4-5 week of disease
On 5-6 week of disease
The diarrheic syndrome and vomiting are the reason:
An infectious-toxic shock
An anaphylactic shock
A dehydrationous shock
A hemorrhagic shock
All right
The hypovolemic shock develops owing to fluid loss at:
A long-term fever
A bleeding (a hemorrhagic shock)
Vomiting and diarrheas
All answers are not true
All answers are true
The hypovolemic shock is:
І degree of dehydration
ІІ degree of dehydration
ІІІ degree of dehydration
ІV degree of dehydration
V degree of dehydration
The peritonitis of the typhoid is accompanied by the onset of:
Stefanskyy's symptom
Mayo-Robson's symptom
Voskresensky symptom
Krymov's symptom
Guarding symptom
The subcompensated dehydrationous shock develops at:
A diastolic and systolic blood pressure boost
A decrease of the systolic blood pressure
A diastolic blood pressure decrease
A diastolic blood pressure boost
A systolic blood pressure boost
The typhoid fever enterorrhagia is characterised with:
A normal pulse
A bradycardia
C. *
D.
E.
687.
A.
B. *
C.
D.
E.
688.
A.
B.
C.
D. *
E.
689.
A. *
B.
C.
D.
E.
690.
A. *
B.
C.
D.
E.
691.
A. *
B.
C.
D.
E.
692.
A. *
B.
C.
D.
E.
693.
A. *
B.
C.
D.
E.
694.
A tachycardia
An alternating pulse
An asystole
Typhoid fever bleedings appears:
On 5-6 week of disease
On 3-4 week of disease
On 1-2 week of disease
On 2 week of disease
On 1 week of disease
Typhoid fever bleedings are accompanied with:
Body temperature decrease and pulse decrease
Body temperature increase and pulse increase
The temperature does not change
Body temperature decrease and pulse increase
Body temperature increase and pulse decrease
What antibiotics preparations of choice of etiotropic therapy at a meningococcal infection.
Benzylpenicillin and it derivatives
Gentamycin
Cefazolin
Sulfolamide
Ciprofloxacin
What antibiotics preparations of choice of etiotropic therapy at a meningococcal infection.
Benzylpenicillin and it derivatives
Gentamycin
Cefazolin
Sulfolamide
Ciprofloxacin
What are the anti epidemic measures in regards to people who were in contact with chicken-pox
patient:
Separation and limit of contacts with others
Vaccination
Use of antibiotics
Disinfection
Does not exist
What are the anti epidemic measures in regards to people who were in contact with chicken-pox
patient:
Separation and limit of contacts with others
Vaccination
Use of antibiotics
Disinfection
Does not exist
What are the anti epidemic measures in regards to people who were in contact with chicken-pox
patient:
Separation and limit of contacts with others
Vaccination
Use of antibiotics
Disinfection
Does not exist
What are the rules at taking of smear material on the discovery of meningococal infection?
A.
B.
C.
D. *
E.
695.
A.
B.
C.
D. *
E.
696.
A.
B.
C.
D. *
E.
697.
A.
B.
C. *
D.
E.
698.
A.
B.
C. *
D.
E.
699.
A.
B.
C. *
D.
E.
700.
A.
B.
C.
D.
E. *
701.
A.
B.
C.
D.
The taken away material at drawing out must not touch only mucus shell of cheeks and tongue
The taken away material at drawing out must not touch only teeth and tongue
The taken away material at drawing out must not touch only teeth, mucus shell of cheeks
The taken away material at drawing out must not touch teeth, mucus shell of cheeks and tongue
The taken away material at drawing out can touch teeth, mucus shell of cheeks and tongue
What are the rules at taking of smear material on the discovery of meningococal infection?
The taken away material at drawing out must not touch only mucus shell of cheeks and tongue
The taken away material at drawing out must not touch only teeth and tongue
The taken away material at drawing out must not touch only teeth, mucus shell of cheeks
The taken away material at drawing out must not touch teeth, mucus shell of cheeks and tongue
The taken away material at drawing out can touch teeth, mucus shell of cheeks and tongue
What clinical form of meningococcal infection more often may happened?
Meningococcemia
Meningitis
Meningoencephalitis
Nasopharengitis
Pneumonia
What group of infectious diseases measles belong to:
Intestinal
Blood
Respiratory
Transmissive
External covers
What group of infectious diseases meningococcal infection belong to:
Intestinal
Blood
Respiratory
Transmissive
External covers
What group of infectious diseases scarlet fever belong to:
Intestinal
Blood
Respiratory
Transmissive
External covers
What is duration of contagious period for a patient with epidemic parotitis?
21 days
First week of illness
First 10 days from the beginning of disease
Whole period of clinical symptoms
First 9 days of disease.
What is duration of contagious period for a patient with epidemic parotitis?
21 days
First week of illness
First 10 days from the beginning of disease
Whole period of clinical symptoms
E. *
702.
A.
B.
C.
D.
E. *
703.
A. *
B.
C.
D.
E.
704.
A. *
B.
C.
D.
E.
705.
A. *
B.
C.
D.
E.
706.
A.
B.
C. *
D.
E.
707.
A.
B.
C. *
D.
E.
708.
A.
B.
C. *
D.
E.
709.
A. *
B.
First 9 days of disease.
What is duration of contagious period for a patient with epidemic parotitis?
21 days
First week of illness
First 10 days from the beginning of disease
Whole period of clinical symptoms
First 9 days of disease.
What is duration period of supervision after ill with scarlet fever?
7 days from time of contact
21 day
Till patient’s rash is present
Till patient is discharged from permanent establishment
Not conducted
What is duration period of supervision after ill with scarlet fever?
7 days from time of contact
21 day
Till patient’s rash is present
Till patient is discharged from permanent establishment
Not conducted
What is duration period of supervision after scarlet fever?
7 days from time of contact
21 days
Till patient’s rash is present
Till patient is discharged from permanent establishment
Not conducted
What is seasonal character of meningococcal infection?
Summer-autumn
Autumn-winter
Winter-spring
Winter
Summer
What is seasonal character of meningococcal infection?
Summer-autumn
Autumn-winter
Winter-spring
Winter
Summer
What is seasonal character of scarlet fever?
Summer-autumn
Autumn-winter
Winter-spring
Winter
Summer
What is taken for serum research for confirmation of meningococcal infection?
Blood
Mucus
C.
D.
E.
710.
A. *
B.
C.
D.
E.
711.
A.
B.
C.
D. *
E.
712.
A.
B.
C.
D. *
E.
713.
A.
B.
C.
D. *
E.
714.
A.
B. *
C.
D.
E.
715.
A.
B.
C.
D. *
E.
716.
A.
B.
C.
D. *
E.
717.
Urine
CSF
Saliva
What is taken for serum research for confirmation of meningococcal infection?
Blood
Mucus
Urine
CSF
Saliva
What is the duration of contagious period for a patient diagnosed with scarlet fever?
10 days from the beginning of illness
Until patient is discharged from the hospital
Until rash is present
Till the 22d day from the beginning of illness
Not contagious
What is the duration of contagious period for a patient diagnosed with scarlet fever?
10 days from the beginning of illness
Until patient is discharged from the hospital
Until rash is present
Till the 22nd day from the beginning of illness
Not contagious
What is the duration of contagious period for a patient diagnosed with scarlet fever?
10 days from the beginning of illness
Until patient is discharged from the hospital
Until rash is present
Till the 22d day from the beginning of illness
Not contagious
What is the duration of quarantine in child's establishment in case of rubella?
11 days
21 day
10 days
No need for quarantine
5 days after isolation of the last child
What is the mechanism of transmission of measles?
Fecal-oral
Contact
Transmissive
Air-drop
Vertical
What is the mechanism of transmission of meningococcal infection?
Fecal-oral
Contact
Transmissive
Air-drop
Vertical
What is the mechanism of transmission of scarlet fever?
A.
B.
C.
D. *
E.
718.
A.
B.
C. *
D.
E.
719.
A.
B.
C. *
D.
E.
720.
A. *
B.
C.
D.
E.
721.
A. *
B.
C.
D.
E.
722.
A.
B.
C.
D. *
E.
723.
A.
B.
C.
D. *
E.
724.
A.
B.
C.
Fecal-oral
Contact
Transmissive
Air-drop
Vertical
What is used as specific prophylaxis in the period of epidemic spreading of meningococcal infection.
Immun globulin
Serum
Vaccine
Anatoxin
Nothing
What is used as specific prophylaxis in the period of epidemic spreading of meningococcal infection.
Immune globulin
Serum
Vaccine
Anatoxin
Nothing
What laboratory methods should be taken to discharge meningitis?
Lumbar puncture
Serologic detection
Urine examination
Coprograma
Biopsy of tissues
What laboratory methods should be taken to discharge meningitis?
Lumbar puncture
Serologic detection
Urine examination
Coprograma
Biopsy of tissues
What measures are conducted in the place of meningococcal infection?
Supervision during 2 weeks
Phagoprophylaxis
Immunization
Bacteriological inspection of contact
Chemoprophylaxis
What measures are conducted in the place of meningococcal infection?
Supervision during 2 weeks
Phagoprophylaxis
Immunization
Bacteriological inspection of contact
Chemoprophylaxis
What measures should be taken in regards to persons, who were in contact with a patient diagnosed
with epidemic parotitis?
Observation after contact people during a maximal length of incubation period
Quorantin in child's establishment
Isolation of people who were in contact with ill from 11th to the 21t day of illness
D.
E. *
725.
A.
B.
C.
D.
E. *
726.
A.
B.
C.
D.
E. *
727.
A.
B.
C.
D.
E. *
728.
A.
B.
C. *
D.
E.
729.
A.
B.
C. *
D.
E.
730.
A.
B.
C. *
D.
E.
731.
A.
B.
C. *
Isolation of children up to 10 years old, who were not ill with epidemic parotitis, for 21 day from a
moment of contact
All above enumerated
What measures should be taken in regards to persons, who were in contact with a patient diagnosed
with epidemic parotitis?
Observation after contact people during a maximal length of incubation period
Quorantine in child's establishment
Isolation of people who were in contact with ill from 11th to the 21th day of illness
Isolation of children up to 10 years old, who were not ill with epidemic parotitis, for 21 day from a
moment of contact
All above enumerated
What measures should be taken in regards to persons, who were in contact with a patient diagnosed
with epidemic parotitis?
Observation after contact people during a maximal length of incubation period
Quorantin in child's establishment
Isolation of people who were in contact with ill from 11th to the 21t day of illness
Isolation of children up to 10 years old, who were not ill with epidemic parotitis, for 21 day from a
moment of contact
All above enumerated
What syndrome may appear in severe meningococcemia?
Paul-Bunnel
Plaut-Vincent
Jarish-Gersgeimer
Gien-Barre
Waterhause-Friedrichsen
What temperature terms is it needed for cultivation of meningococcal on artificial mediums?
23-40 °C
35-43 °C
35-37 °C
23-35 °C
37-39 °C
What temperature terms is it needed for cultivation of meningococcal on artificial mediums?
23-40 °C
35-43 °C
35-37 °C
23-35 °C
37-39 °C
What type of a diarrhoeia is typical for a salmonellosis?
Osmotic
Exudative
Secretory
Mixed
Toxic
When does the laboratory give the results of bacteriological examination of smear from throat?
On 2th days
On 3th days
On 4th days
D.
E.
732.
A.
B.
C. *
D.
E.
733.
A.
B.
C.
D.
E. *
734.
A.
B.
C.
D.
E. *
735.
A.
B.
C.
D.
E. *
736.
A.
B.
C.
D.
E. *
737.
A.
B.
C.
D.
E. *
On 5th days
On 6th days
When does the laboratory give the results of bacteriological examination of smear from throat?
On 2th days
On 3th days
On 4th days
On 5th days
On 6th days
When patient refered to the doctor with such complaints: prodromal respiratory illness, sore throat,
fever, headache, stiff neck, vomiting, confusion, irritability what is previous diagnosis. What main
methods can confirm the diagnosis?
Epidemic typhys. ELISA-test
Hemorrhagic fever. ELISA-test
Leptospirosis. Lumbar puncture.
Typhoid fever. ELISA-test
Meningococcal infection. Lumbar puncture
Which preparation has a bacteriostatic action, and is more expedient to begin etiotropic therapy in the
case of infectious toxic shock.
From benzylpenicillin and its derivatives
From rovamicin
From gentamycin
From ciprofloxacin
From ceftriaxon
Which preparation has a bacteriostatic action, and is more expedient to begin etiotropic therapy in the
case of infectious toxic shock.
From benzylpenicillin and its derivatives
From ciprofloxacin
From gentamycin
From acyclovir
From chloramphenicol
Wich of these symptoms are often present in patients with meningitis?
Algor, high temperature, headache
Profuse watery diarrhea, vomiting, dehydratation, muscular cramps
Abdominal pain, diarrhea, constipation, flatulence
Headache, dry cough, algor
Prodromal respiratory illness or sore throat, fever, headache, stiff neck, vomiting, confusion,
irritability
Witch of these symptoms are often present in patients with meningitis?
Algor, high temperature, headache
Profuse watery diarrhea, vomiting, dehydratation, muscular cramps
Abdominal pain, diarrhea, constipation, flatulence
Headache, dry cough, algor
Prodromal respiratory illness or sore throat, fever, headache, stiff neck, vomiting, confusion,
irritability